Огород имеет вид прямоугольника длина которого 340 м: огород имеет вид прямоугольника, длина которого 340 м, а ширина 220 м. какие размеры будет

Содержание

Страница 10 №21-28 ГДЗ к учебнику «Математика» 6 класс Никольский, Потапов, Решетников, Шевкин

Ответы к разделу 1.2 учебника. Масштаб

Задание 21. Что показывает численный масштаб:
а) 1 : 100;
б) 1 : 1000;
в) 1 : 20000.

Ответы 7 гуру

а) Расстояние на карте в 100 раз меньше чем на местности.
б) Ответ: расстояние на карте в 1000 раз меньше чем на местности.
в) Ответ: расстояние на карте в 20000 раз меньше чем на местности.

Задание 22. Определите численный масштаб, если известно, что 1 см на плане (географической карте) изображает отрезок длиной:
а) 10 см;
б) 50 см;
в) 6 дм;
г) 10 м;
д) 100 м;
е) 1 км;
ж) 6 км;
з) 10 км.

Ответы

а)  1 
    10
б)  1 
    50
в)  1 
    60
г)    1  
   1000
д)    1    
   10000
е)     1     
    100000
 ж)     1   
    600000
з)         1    
    1000000

Задание 23. Расстояние между двумя городами равно 200 км. Определите расстояние между изображениями этих городов на карте, если численный масштаб карты равен:
а) 1 : 1 000 000;
б) 1 : 200 000;
в) 1/5000000.

Решение

а) 200 км * 100000 = 20000000 см.
20000000 см * 1 : 1000000 = 20 см
Ответ: 20см

б)  200 км * 100000 = 20000000 см.
20000000 см * 1 : 200000 = 100 см
Ответ: 100 см

в) 200 км * 100000 = 20000000 см.
20000000 см * 1 = 20000000 см = 4 см
5000000 5000000
Ответ: 4 см.

Задание 24. Масштаб карты равен 1/50000. Определите расстояние на местности, если на карте оно равно:
а) 1 см;
б) 5 см;
в) 22 см;
г) 37 мм;
д) 1 целая и 1/5 дм;
е) 146 мм.

Решение

Задание 25. План комнаты имеет вид прямоугольника со сторонами 40 мм и 31 мм. Определите длину и ширину комнаты, если численный масштаб плана равен 1 : 200.

Решение

Задание 26. Огород имеет вид прямоугольника, длина которого 340 м, а ширина 220 м. Какие размеры будет иметь изображение этого огорода на плане, выполненном в масштабе  1/500  ?

Решение

Задание 27. Прямоугольник со сторонами 12 см и 6 см изображает на плане поле, занятое под овёс. Определите масштаб плана, если большая сторона поля имеет длину 360 м. Определите меньшую сторону поля.

Решение

Задание 28. Используя план местности (рис. 2) определите:
а) расстояние от А до B;
б) расстояние от А и от B до моста через реку;
в) расстояние от B до смешанного леса.
Масштаб 1:10000

Решение

 

6 класс. Математика. Никольский. Учебник. Ответы к стр. 10

Отношения, пропорции, проценты

Масштаб

Ответы к стр. 10

21. Что показывает численный масштаб:
а) 1 : 100; б) 1 : 1000; в) 1 : 20 000.

а) расстояние на карте в 100 раз меньше расстояния на местности;
б) расстояние на карте в 1000 раз меньше расстояния на местности;
в) расстояние на карте в 20 000 раз меньше расстояния на местности.

22. Определите численный масштаб, если известно, что 1 см на плане (географической карте) изображает отрезок длиной:
а) 10 см; б) 50 см; в) 6 дм; г) 10 м;
д) 100 м; е) 1 км; ж) 6 км; з) 10 км.

а) 1 : 10;
б) 1 : 50;
в) 1 : 60, так как 6 дм = 60 см;
г) 1 : 1000, так как 10 м = 1000 см;
д) 1 : 10 000, так как 100 м = 10 000 см;
е) 1 : 100 000, так как 1 км = 100 000 см;
ж) 1 : 600 000, так как 6 км = 600 000 см;
з) 1 : 1 000 000, так как 10 км = 1 000 000 см.

23. Расстояние между двумя городами равно 200 км. Определите расстояние между изображениями этих городов на карте, если численный масштаб карты равен:
а) 1 : 1 000 000; б) 1 : 200 000; в) 1/5000000.

а) 200 км = 20 000 000 см
1 : 1 000 000 = 1/1000000
20 000 000 см • 1/1000000 = 20000000/1000000 см = 20 см
О т в е т: 20 см.

б) 200 км = 20 000 000 см
1 : 200 000 = 1/200000
20 000 000 см • 1/200000 = 20000000/200000 см = 100 см
О т в е т: 100 см.

в) 200 км = 20 000 000 см
20000000 см • 1/5000000 = 20000000/5000000 см = 4 см
О т в е т: 4 см.

24. Масштаб карты равен 1/50000. Определите расстояние на местности, если на карте оно равно:
а) 1 см; б) 5 см; в) 22 см; г) 37 мм; д) 1 1/5 дм; е) 146 мм.

а) 1 см : 1/50000 = 1 см • 50000/1 = 50 000 см = 500 м
О т в е т: 500 м.

б) 5 см : 1/50000 = 5 см • 50000/1 = 250 000 см = 2500 м = 2 км 500 м
О т в е т: 2 км 500 м.

в) 22 см : 1/50000 = 22 см • 50000/1 = 1 100 000 см = 11 000 м = 11 км
О т в е т: 11 км.

г) 37 мм = 37/10 см
37/10 см : 1/50000 = 37/10 см • 50000/1 = 37•50000/10 см = 37 см•5000/1 = 185 000 см = 1850 м = 1 км 850 м
О т в е т: 1 км 850 м.

д) 1 1/5 дм = 6/5 дм = 6/5 • 10 см = 6•10/5 см = 60/5 см = 12 см
12 см : 1/50000 = 12 см • 50000/1 = 600 000 см = 6000 м = 6 км
О т в е т: 6 км.

е) 146 мм = 146/10 см = 73/5 см
73/5 см : 1/50000 = 73/5 см • 50000/1 = 73•50000/5 см = 73 см•10000/1 = 730 000 см = 7300 м = 7 км 300 м
О т в е т: 7 км 300 м.

25. План комнаты имеет вид прямоугольника со сторонами 40 мм и 31 мм. Определите длину и ширину комнаты, если численный масштаб плана равен 1 : 200.

1 : 200 = 1/200
Длина α = 40 мм : 1/200 = 40 мм • 200/1 = 8000 мм = 800 см = 8 м
Ширина b = 31 мм : 1/200 = 31 мм • 200/1 = 6200 мм = 620 см = 6 м 20 см
О т в е т: длина α = 8 м, ширина b = 6 м 20 см.

26. Огород имеет вид прямоугольника, длина которого 340 м, а ширина 220 м. Какие размеры будет иметь изображение этого огорода на плане, выполненном в масштабе 1 : 500?

1 : 500 = 1/500
Длина α = 340 м • 1/500 = 340/500 м = 34/50 м = 17/25 м = 1700/25 см = 68 см
Ширина b = 220 м • 1/500 = 220/500 м = 22/50 м = 11/25 м = 1100/25 см = 44 см
Ответ: длина изображение на плане α = 68 см, ширина изображения на плане b = 44 см.

27. Прямоугольник со сторонами 12 см и 6 см изображает на плане поле, занятое под овёс. Определите масштаб плана, если бóльшая сторона поля имеет длину 360 м. Определите меньшую сторону поля.

Масштаб М = 12 см : 360 м = 12 см : 36 000 см = 12/36000 = 1/3000 = 1 : 3000
Ширина b = 6 см 1/3000 = 6 см • 3000/1 = 18 000 см = 180 м
О т в е т: масштаб 1 : 3000, ширина поля b = 180 м.

28. Используя план местности (рис. 2) определите:
а) расстояние от А до B;
б) расстояние от А и от B до моста через реку;
в) расстояние от B до смешанного леса.

 

а) 1 : 10 000 = 1/10000
расстояние на плане от А до B примерно 70 мм = 7 см
расстояние на местности от А до B: s = 7 см : 1/10000 = 7 см • 10000/1  = 70 000 см = 700 м.
О т в е т: от А до В 700 м.

б) 1 : 10000 = 1/10000
расстояние на плане от А до моста через реку примерно 20 мм = 2 см
расстояние на плане от В до моста через реку примерно 50 мм = 5 см
расстояние на местности от А до моста через реку: s = 2 см : 1/10000 = 2 см • 10000/1 = 20 000 см = 200 м
расстояние на местности от В до моста через реку: s = 5 см : 1/10000 = 5 см • 10000/1 = 50 000 см = 500 м.
О т в е т: от А до моста 200 м, от В до моста 500 м.

в) 1 : 10000 = 1/10000
расстояние на плане от B до смешанного леса примерно 15 мм = 15/10 см = 3/2 см
расстояние на местности от B до смешанного леса: s3/2 : 1/10000 = 3/2 • 10000/1 = 30000/2 = 15 000 см = 150 м.
О т в е т: от В до смешанного леса 150 м.

Ответы по математике. 6 класс. Учебник. Никольский С.М., Потапов М.К., Решетников Н.Н., Шевкин А.В.

Математика. 6 класс

ГДЗ По Математике 6 Класс Задание 26 – Telegraph

➡➡➡ ПОДРОБНЕЕ ЖМИТЕ ЗДЕСЬ!

ГДЗ По Математике 6 Класс Задание 26

ГДЗ (готовое домашние задание из решебника) на Номер №26 по учебнику Математика . 6 класс : учебник для общеобразовательных учреждений / Н . Я . Виленкин, В . И . Жохов, А . С . Чесноков, С . И . Шварцбурд . — 30-е изд ., стер . — М . : Мнемозина, -2020 

ГДЗ по математике для 6 класса — Герасимов . Авторы .  Размещенные на данной странице готовые домашние задания помогут освоить программу по математике 6 класса, разобрать те моменты, которые были не до конца поняты на уроке, и выполнить на отлично все упражнения . 

В шестом классе математика становится не только сложнее, но и интереснее . За все время обучения ребята усвоили основы, и теперь перед  Чтобы не пугаться трудностей и не отставать по программе, советуем использовать ГДЗ по математике для 6 класса, — она подскажет как . . 

Комментарий: Давайте вспомним определение слова делитель, это определения вы всегра можете найти в вашем учебнике по математике для шестых классов на странице номер четыре . 

Задача №26 , ГДЗ по математике за 6 класс к учебнику Виленкина с подробным решением . 

Готовые домашние задания по математике 6 класса под авторством Герасимов В . Д . и Пирютко О .Н . 2019-2020 .  Преимущества ГДЗ по математике . Качественный решебник также может похвастаться и таким разделом, как объяснения к выполнению заданий . 

Решебник, готовые домашние задания (ГДЗ ) по математике для учащихся 6 класса, авторов Герасимов В . Д ., Пирютко О . Н .  В 6 классе ты заканчиваешь изучать математику . Дальше — алгебра и геометрия . На протяжении учебного года ты получишь знания, которые используются . . 

Герасимов В .Д ., Пирютко О .Н . Решебник (ГДЗ ) по Математике за 6 (шестой ) класс авторы: Герасимов, Пирютко издательство Образование и воспитание, 2019 год .
ГДЗ учебник по математике 6 класс Мерзляк . авторы: А .Г . Мерзляк, В .Б . Полонский, М .С . Якир . издательство: Вентана-Граф, 2019   ГДЗ — Готовые Домашние Задания . 

ГДЗ по математике 6 класс Виленкин учебник . номер — 26 (26) . Авторы : Н .Я . Виленкин, В .И . Жохов, А .С . Чесноков, С .И . Шварцбурд . Издательство: Мнемозина -2020 . Тип книги: Учебник . 

Подробное решение задачи по математике № 26 . Условие задачи . Найти все делители числа 30 и запишите их в порядке возрастания . Задача по математике 6 класс № 26 . 

Решенное задание Упр .26 Часть 2 из учебника Виленкин Жохов 6 класс ФГОС по математике бесплатно с пояснениями .  26 . На рисунке 12,а изображён цилиндр . Сверху и снизу цилиндр ограничен кругами, которые называются основаниями цилиндра . 

Видео решение к номеру 26 по математике за 6 класс , авторов Н .Я . Виленкин, В .И . Жохов, А .С . Чесноков, С .И . Шварцбурд . Более подробное гдз к этому заданию . .
Решение задания номер 26 .  ГДЗ , математика , 6 класс — Зубарева, Мордкович — онлайн решебник . 

Задание № 26 — ГДЗ по математике 6 класс (Никольский, Потапов) .  ✅ Задание № 26 Огород имеет вид прямоугольника, длина которого 340 м, а ширина 220 м . Какие размеры будет иметь изображение этого огорода на плане, выполненном в масштабе 1/500? 

ГДЗ (готовое домашние задание из решебника) на Номер №26 по учебнику Математика . 6 класс : учебник для общеобразовательных учреждений / Н . Я . Виленкин, В . И . Жохов, А . С . Чесноков, С . И . Шварцбурд . — 30-е изд ., стер . — М . : Мнемозина, -2020 

ГДЗ по математике для 6 класса — Герасимов . Авторы .  Размещенные на данной странице готовые домашние задания помогут освоить программу по математике 6 класса, разобрать те моменты, которые были не до конца поняты на уроке, и выполнить на отлично все упражнения . 

В шестом классе математика становится не только сложнее, но и интереснее . За все время обучения ребята усвоили основы, и теперь перед  Чтобы не пугаться трудностей и не отставать по программе, советуем использовать ГДЗ по математике для 6 класса, — она подскажет как . . 

Комментарий: Давайте вспомним определение слова делитель, это определения вы всегра можете найти в вашем учебнике по математике для шестых классов на странице номер четыре . 

Задача №26 , ГДЗ по математике за 6 класс к учебнику Виленкина с подробным решением . 

Готовые домашние задания по математике 6 класса под авторством Герасимов В . Д . и Пирютко О .Н . 2019-2020 .  Преимущества ГДЗ по математике . Качественный решебник также может похвастаться и таким разделом, как объяснения к выполнению заданий . 

Решебник, готовые домашние задания (ГДЗ ) по математике для учащихся 6 класса, авторов Герасимов В . Д ., Пирютко О . Н .  В 6 классе ты заканчиваешь изучать математику . Дальше — алгебра и геометрия . На протяжении учебного года ты получишь знания, которые используются . . 

Герасимов В .Д ., Пирютко О .Н . Решебник (ГДЗ ) по Математике за 6 (шестой ) класс авторы: Герасимов, Пирютко издательство Образование и воспитание, 2019 год .
ГДЗ учебник по математике 6 класс Мерзляк . авторы: А .Г . Мерзляк, В .Б . Полонский, М .С . Якир . издательство: Вентана-Граф, 2019   ГДЗ — Готовые Домашние Задания . 

ГДЗ по математике 6 класс Виленкин учебник . номер — 26 (26) . Авторы : Н .Я . Виленкин, В .И . Жохов, А .С . Чесноков, С .И . Шварцбурд . Издательство: Мнемозина -2020 . Тип книги: Учебник . 

Подробное решение задачи по математике № 26 . Условие задачи . Найти все делители числа 30 и запишите их в порядке возрастания . Задача по математике 6 класс № 26 . 

Решенное задание Упр .26 Часть 2 из учебника Виленкин Жохов 6 класс ФГОС по математике бесплатно с пояснениями .  26 . На рисунке 12,а изображён цилиндр . Сверху и снизу цилиндр ограничен кругами, которые называются основаниями цилиндра . 

Видео решение к номеру 26 по математике за 6 класс , авторов Н .Я . Виленкин, В .И . Жохов, А .С . Чесноков, С .И . Шварцбурд . Более подробное гдз к этому заданию . .
Решение задания номер 26 .  ГДЗ , математика , 6 класс — Зубарева, Мордкович — онлайн решебник . 

Задание № 26 — ГДЗ по математике 6 класс (Никольский, Потапов) .  ✅ Задание № 26 Огород имеет вид прямоугольника, длина которого 340 м, а ширина 220 м . Какие размеры будет иметь изображение этого огорода на плане, выполненном в масштабе 1/500? 

Решебник Английский 6 Класс Фокус
ГДЗ По Немецкому Языку 8 Класс Горизонты
ГДЗ По Английскому Языку 5 Класса Ваулина
ГДЗ По Окружающему Миру 1
ГДЗ Разумовская 9 Класс 2009
ГДЗ Афанасьева 6 Класс Лексико
ГДЗ По Русскому 10 Класс Вентана Граф
ГДЗ По Русской Речи 9 Класс
Где Посмотреть ГДЗ Бесплатно
ГДЗ По Учебнику Русского Языка Третьего
ГДЗ Русский 5 Класс Тетрадь Янченко
ГДЗ По Матем 4кл Петерсон
ГДЗ Соловейчик 3 Класс
ГДЗ По Матем 3к Моро
Решебник По Математике 1х Класс Алимов
Готовое Домашнее Задание 3 Класса Рудницкая
ГДЗ По Русскому 2 Класс 2часть
ГДЗ По Алгебре 7 Класс Задачник Мнемозина
ГДЗ Русский 4 Класс Канакин
ГДЗ Английский 4 Класс Баранова Тетрадь
ГДЗ Дидактический 8 Класс Макарычев
Решебник Перышкин 7 9
Информатика 3 Класс Рабочая Рудченко ГДЗ
Журавлев 7 Класс ГДЗ
ГДЗ По Алгебре 7 Класс Бунимович Кузнецова
ГДЗ По Русскому 9 Класс Богданова 1
ГДЗ По Алгебре 7 Класс Теляковского 2005
Решебник Русский Язык 2 Класс Учебник Соловейчик
Решебник Новошинский 11 Класс
Решебник ГДЗ По Английскому Языку 7 Класс
Решебник 5 Математики Виленкин Скачать
ГДЗ По Физике 7 Класс Перышкин Фгос
ГДЗ Учебник 1
ГДЗ По Геометрии Дидактический
Решебник Аргинская 2 Класс
ГДЗ По Английскому Языку 8 Биболетова
Решебник По Геометрии 7 Класс Дидактические
ГДЗ Ответы Физика
ГДЗ По Математике 2 Класс Автор Волкова
ГДЗ По Математике 4 Класс Граф
ГДЗ По Геометрии 7 Класс Атанасян Мегарешеба
ГДЗ Английский 9 Класс Афанасьева Углубленный Уровень
ГДЗ История Нового Времени 8 Класс Учебник
Каленчук 4 Класс 1 Часть Учебник ГДЗ
ГДЗ По Математике 6 Класс Виленкин 58
ГДЗ По Алгебре 11 Класс Колягин 2015
ГДЗ Математика 2 Класс Стр 64
ГДЗ По Немецкому Языку 8 Класс Горизонт
ГДЗ По Биологии 6 Романова
ГДЗ Англ Сборник Упражнений 4

ГДЗ По Русскому Учебник 6 Класс Бабайцева

ГДЗ По Биологии 9

ГДЗ По Физике Рымкевич 8

Гдз По Окружающему Часть

География России 9 Класс ГДЗ

треугольников, прямоугольников и теорема Пифагора — элементарная алгебра

Цели обучения

К концу этого раздела вы сможете:

  • Решать приложения, используя свойства треугольников
  • Используйте теорему Пифагора
  • Решение приложений с использованием свойств прямоугольника

Решение приложений, используя свойства треугольников

В этом разделе мы будем использовать некоторые общие геометрические формулы. Мы адаптируем нашу стратегию решения проблем, чтобы мы могли решать геометрические приложения.Формула геометрии назовет переменные и даст нам уравнение для решения. Кроме того, поскольку все эти приложения будут включать в себя какие-то формы, большинство людей сочтут полезным нарисовать фигуру и пометить ее с заданной информацией. Мы включим это в первый шаг стратегии решения проблем для геометрических приложений.

Приложения для решения геометрии.

  1. Прочтите задачу и убедитесь, что все слова и идеи понятны. Нарисуйте фигуру и напишите на ней указанную информацию.
  2. Укажите , что мы ищем.
  3. Этикетка то, что мы ищем, выбирая переменную для его представления.
  4. Переведите в уравнение, написав соответствующую формулу или модель для ситуации. Подставьте в данную информацию.
  5. Решите уравнение, используя хорошие методы алгебры.
  6. Проверьте ответ, подставив его обратно в уравнение, решенное на шаге 5, и убедившись, что он имеет смысл в контексте проблемы.
  7. Ответьте на вопрос полным предложением.

Мы начнем с геометрических приложений, изучив свойства треугольников. Давайте рассмотрим некоторые основные факты о треугольниках. Треугольники имеют три стороны и три внутренних угла. Обычно каждая сторона помечена строчной буквой, которая соответствует прописной букве противоположной вершины.

Множественное число слова вершина составляет вершины . У всех треугольников по три вершины.Треугольники названы по их вершинам: Треугольник на (Рисунок) называется

.

Треугольник ABC имеет вершины A, B и C. Длины сторон равны a, b и c.

Три угла треугольника связаны особым образом. Сумма их мер равна. Обратите внимание, что мы читаем как «мера угла А. » Итак, на (рисунок),

Поскольку периметр фигуры равен длине ее границы, периметр фигуры равен сумме длин трех ее сторон.

Чтобы найти площадь треугольника, нам нужно знать его основание и высоту. Высота — это линия, которая соединяет основание с противоположной вершиной и составляет угол с основанием. Нарисуем еще раз, а теперь покажем высоту х . См. (Рисунок).

Формула для площади: где b — основание, а h — высота.

Свойства треугольника

для

Размеры угла:

  • Сумма углов треугольника равна
  • .

Периметр:

  • Периметр — это сумма длин сторон треугольника.

Площадь:

  • Площадь треугольника равна половине основания, умноженной на высоту.

Размеры двух углов треугольника — 55 и 82 градуса. Найдите размер третьего угла.

Размеры двух углов треугольника — 31 и 128 градус. Найдите размер третьего угла.

Размеры двух углов треугольника — 49 и 75 градусов. Найдите размер третьего угла.

Периметр треугольного сада составляет 24 фута. Длина двух сторон четыре фута и девять футов. Какова длина третьей стороны?

Периметр треугольного сада составляет 48 футов. Длина двух сторон 18 футов и 22 фута. Какова длина третьей стороны?

Длина двух сторон треугольного окна составляет семь футов пять футов. По периметру 18 футов. Какова длина третьей стороны?

Площадь треугольного церковного окна — 90 квадратных метров.База окна 15 метров. Какая высота окна?

Площадь треугольной картины составляет 126 квадратных дюймов. База 18 дюймов. Какая высота?

Треугольная дверь палатки имеет площадь 15 квадратных футов. Высота пять футов. Что такое база?

Свойства треугольника, которые мы использовали до сих пор, применимы ко всем треугольникам. Теперь мы рассмотрим один конкретный тип треугольника — прямоугольный треугольник. Прямоугольный треугольник имеет один угол, который мы обычно отмечаем маленьким квадратом в углу.

Прямой треугольник

Прямоугольный треугольник имеет один угол, который часто отмечается квадратом в вершине.

Измеряет один угол прямоугольного треугольника. Какова мера третьего угла?

Размер одного угла прямоугольного треугольника Какова мера другого малого угла?

Размер одного угла прямоугольного треугольника Какова мера другого малого угла?

В примерах, которые мы видели до сих пор, мы могли нарисовать фигуру и пометить ее сразу после прочтения задачи.В следующем примере нам нужно будет определить один угол через другой. Мы будем ждать, чтобы нарисовать фигуру, пока не напишем выражения для всех искомых углов.

Размер одного угла прямоугольного треугольника на 20 градусов больше меры наименьшего угла. Найдите размеры всех трех углов.

Размер одного угла прямоугольного треугольника на 50 ° больше, чем размер наименьшего угла. Найдите размеры всех трех углов.

Размер одного угла прямоугольного треугольника на 30 ° больше, чем размер наименьшего угла.Найдите размеры всех трех углов.

Используйте теорему Пифагора

Мы узнали, как соотносятся друг с другом размеры углов треугольника. Теперь мы узнаем, как длины сторон соотносятся друг с другом. Важное свойство, которое описывает соотношение между длинами трех сторон прямоугольного треугольника, называется теоремой Пифагора. Эта теорема использовалась во всем мире с древних времен. Он назван в честь греческого философа и математика Пифагора, жившего около 500 г. до н.э.

Прежде чем мы сформулируем теорему Пифагора, нам нужно ввести некоторые термины для сторон треугольника. Помните, что у прямоугольного треугольника есть угол, отмеченный маленьким квадратом в углу. Сторона треугольника, противоположная углу, называется гипотенузой , а каждая из других сторон называется катетом .

Теорема Пифагора говорит, как длины трех сторон прямоугольного треугольника соотносятся друг с другом. Он утверждает, что в любом прямоугольном треугольнике сумма квадратов длин двух катетов равна квадрату длины гипотенузы.В символах мы говорим: в любом прямоугольном треугольнике, где — длины катетов, а — длина гипотенузы.

Написание формулы в каждом упражнении и произнесение ее вслух во время написания может помочь вам запомнить теорему Пифагора.

Теорема Пифагора

В любом прямоугольном треугольнике

, где a и b — длина катетов, c — длина гипотенузы.

Чтобы решить упражнения, в которых используется теорема Пифагора, нам нужно найти квадратные корни. Мы использовали обозначения и определение:

Если то за

Например, мы обнаружили, что это 5, потому что

Поскольку теорема Пифагора содержит возведенные в квадрат переменные, чтобы найти длину стороны прямоугольного треугольника, нам придется использовать квадратные корни.

Используйте теорему Пифагора, чтобы найти длину гипотенузы, показанной ниже.

Используйте теорему Пифагора, чтобы найти длину гипотенузы в треугольнике, показанном ниже.

Используйте теорему Пифагора, чтобы найти длину гипотенузы в треугольнике, показанном ниже.

Используйте теорему Пифагора, чтобы найти длину ноги, показанной ниже.

Используйте теорему Пифагора, чтобы найти длину катета в треугольнике, показанном ниже.

Используйте теорему Пифагора, чтобы найти длину катета в треугольнике, показанном ниже.

Джон ставит основание 13-футовой лестницы в пяти футах от стены своего дома, как показано ниже. Как далеко до стены поднимается лестница?

Рэнди хочет прикрепить 17-футовую гирлянду фонарей к вершине 15-футовой мачты своей парусной лодки, как показано ниже. На каком расстоянии от основания мачты он должен прикрепить конец световой струны?

Решение приложений с использованием свойств прямоугольника

Возможно, вы уже знакомы со свойствами прямоугольников.Прямоугольники имеют четыре стороны и четыре прямых угла. Противоположные стороны прямоугольника имеют одинаковую длину. Мы называем одну сторону прямоугольника длиной L , а его прилегающую сторону шириной W .

Расстояние вокруг этого прямоугольника равно или Это периметр прямоугольника P .

А как насчет площади прямоугольника? Представьте себе прямоугольный коврик длиной 2 фута и шириной 3 фута. Его площадь составляет 6 квадратных футов. На рисунке шесть квадратов.

Площадь равна длине, умноженной на ширину.

Формула площади прямоугольника

Свойства прямоугольников

Прямоугольник имеет четыре стороны и четыре прямых угла.

Длины противоположных сторон равны.

Периметр прямоугольника равен сумме удвоенной длины и удвоенной ширины.

Площадь прямоугольника равна произведению длины и ширины.

Длина прямоугольника 32 метра, ширина 20 метров.Какой периметр?

Длина прямоугольника составляет 120 ярдов, а ширина — 50 ярдов. Какой периметр?

Длина прямоугольника 62 фута, ширина 48 футов. Какой периметр?

Площадь прямоугольной комнаты составляет 168 квадратных футов. Длина 14 футов. Какая ширина?

Площадь прямоугольника составляет 598 квадратных футов. Длина 23 фута. Какая ширина?

Ширина прямоугольника 21 метр.Площадь 609 кв. Какая длина?

Найдите длину прямоугольника с периметром 50 дюймов и шириной 10 дюймов.

Найдите длину прямоугольника с периметром 80 и шириной 25.

Найдите длину прямоугольника с периметром 30 и шириной 6.

Мы решили задачи, в которых задавалась длина или ширина, а также периметр или площадь; Теперь мы научимся решать задачи, в которых ширина определяется длиной.Мы будем ждать, чтобы нарисовать фигуру, пока не напишем выражение для ширины, чтобы мы могли пометить одну сторону этим выражением.

Ширина прямоугольника на два фута меньше его длины. Периметр — 52 фута. Найдите длину и ширину.

Ширина прямоугольника на семь метров меньше его длины. Периметр — 58 метров. Найдите длину и ширину.

Длина прямоугольника на восемь футов больше ширины. Периметр — 60 футов.Найдите длину и ширину.

Длина прямоугольника на четыре сантиметра больше ширины в два раза. По периметру 32 сантиметра. Найдите длину и ширину.

Длина прямоугольника в восемь раз больше ширины в два раза. Периметр равен 64. Найдите длину и ширину.

Ширина прямоугольника в шесть раз меньше его длины в два раза. Периметр равен 18. Найдите длину и ширину.

Периметр прямоугольного бассейна составляет 150 футов.Длина на 15 футов больше ширины. Найдите длину и ширину.

Периметр прямоугольного бассейна составляет 200 футов. Длина на 40 футов больше ширины. Найдите длину и ширину.

Длина прямоугольного сада на 30 ярдов больше ширины. Периметр 300 ярдов. Найдите длину и ширину.

Практика ведет к совершенству

Решение приложений с использованием свойств треугольника

В следующих упражнениях решите, используя свойства треугольника.

Размеры двух углов треугольника — 26 и 98 градусов. Найдите размер третьего угла.

Размеры двух углов треугольника 61 и 84 градуса. Найдите размер третьего угла.

Размеры двух углов треугольника — 105 и 31 градус. Найдите размер третьего угла.

Размеры двух углов треугольника — 47 и 72 градуса. Найдите размер третьего угла.

Периметр треугольного бассейна — 36 ярдов.Длина двух сторон составляет 10 ярдов и 15 ярдов. Какова длина третьей стороны?

Треугольный двор имеет периметр 120 метров. Длина двух сторон 30 метров и 50 метров. Какова длина третьей стороны?

Если треугольник имеет стороны 6 футов и 9 футов, а периметр равен 23 футам, какова длина третьей стороны?

Если треугольник имеет стороны 14 и 18 см, а периметр равен 49 см, какова длина третьей стороны?

Треугольный флаг имеет основание одна ножка и высоту 1.5 футов. Какая у него площадь?

Треугольное окно имеет основание восемь футов и высоту шесть футов. Какая у него площадь?

Что такое основание треугольника площадью 207 квадратных дюймов и высотой 18 дюймов?

Какова высота треугольника с площадью 893 квадратных дюйма и основанием 38 дюймов?

Один угол прямоугольного треугольника составляет 33 градуса. Какова мера другого малого угла?

Один угол прямоугольного треугольника составляет 51 градус.Какова мера другого малого угла?

Один угол прямоугольного треугольника составляет 22,5 градуса. Какова мера другого малого угла?

Один угол прямоугольного треугольника составляет 36,5 градуса. Какова мера другого малого угла?

Периметр треугольника составляет 39 футов. Одна сторона треугольника на один фут длиннее второй стороны. Третья сторона на два фута длиннее второй. Найдите длину каждой стороны.

Периметр треугольника составляет 35 футов.Одна сторона треугольника на пять футов длиннее второй. Третья сторона на три фута длиннее второй. Найдите длину каждой стороны.

Одна сторона треугольника вдвое короче. Третья сторона на пять футов больше самой короткой. Периметр — 17 футов. Найдите длины всех трех сторон.

Одна сторона треугольника в три раза длиннее самой короткой стороны. Третья сторона на три фута больше самой короткой. Периметр — 13 футов.Найдите длины всех трех сторон.

Два меньших угла прямоугольного треугольника имеют равные размеры. Найдите размеры всех трех углов.

Размер наименьшего угла прямоугольного треугольника на 20 ° меньше размера следующего большего угла. Найдите размеры всех трех углов.

Углы в треугольнике таковы, что один угол в два раза больше наименьшего угла, а третий угол в три раза больше наименьшего угла.Найдите размеры всех трех углов.

Углы в треугольнике таковы, что один угол на 20 ° больше наименьшего угла, а третий угол в три раза больше наименьшего угла. Найдите размеры всех трех углов.

Используйте теорему Пифагора

В следующих упражнениях используйте теорему Пифагора, чтобы найти длину гипотенузы.

В следующих упражнениях используйте теорему Пифагора, чтобы найти длину ноги.При необходимости округлите до ближайшей десятой.

В следующих упражнениях решите, используя теорему Пифагора. При необходимости с точностью до десятых долей.

13-футовая гирлянда огней будет прикреплена к вершине 12-футовой стойки для праздничной демонстрации, как показано ниже. На каком расстоянии от основания столба должен быть закреплен конец гирлянды?

Пэм хочет повесить плакат на двери своего гаража, как показано ниже, чтобы поздравить сына с окончанием колледжа.Дверь гаража имеет высоту 12 футов и ширину 16 футов. Какой длины должен быть баннер, чтобы подходить к воротам гаража?

Чи планирует проложить дорожку из брусчатки в своем цветнике, как показано ниже. Цветник представляет собой квадрат со стороной 10 футов. Какой будет длина пути?

Брайан одолжил 20-футовую удлинительную лестницу, чтобы использовать ее, когда красит свой дом. Если он установит основание лестницы на расстоянии 6 футов от дома, как показано ниже, насколько высоко поднимется верх лестницы?

Решение приложений с использованием свойств прямоугольника

В следующих упражнениях решите, используя свойства прямоугольника.

Длина прямоугольника составляет 85 футов, а ширина — 45 футов. Какой периметр?

Длина прямоугольника составляет 26 дюймов, а ширина — 58 дюймов. Какой периметр?

Прямоугольная комната 15 футов шириной и 14 футов длиной. Каков его периметр?

Подъездная дорога имеет форму прямоугольника 20 футов шириной и 35 футов длиной. Каков его периметр?

Площадь прямоугольника 414 квадратных метров. Длина 18 метров.Какая ширина?

Площадь прямоугольника 782 квадратных сантиметра. Ширина 17 сантиметров. Какая длина?

Ширина прямоугольного окна 24 дюйма. Площадь — 624 квадратных дюйма. Какая длина?

Длина прямоугольного плаката составляет 28 дюймов. Площадь составляет 1316 квадратных дюймов. Какая ширина?

Найдите длину прямоугольника с периметром 124 и шириной 38.

Найдите ширину прямоугольника с периметром 92 и длиной 19.

Найдите ширину прямоугольника с периметром 16,2 и длиной 3,2.

Найдите длину прямоугольника с периметром 20,2 и шириной 7,8.

Длина прямоугольника на девять дюймов больше ширины. По периметру 46 дюймов. Найдите длину и ширину.

Ширина прямоугольника на восемь дюймов больше его длины. По периметру 52 дюйма. Найдите длину и ширину.

Периметр прямоугольника 58 метров.Ширина прямоугольника на пять метров меньше длины. Найдите длину и ширину прямоугольника.

Периметр прямоугольника 62 фута. Ширина на семь футов меньше длины. Найдите длину и ширину.

Ширина прямоугольника на 0,7 метра меньше длины. Периметр прямоугольника 52,6 метра. Найдите размеры прямоугольника.

Длина 13,5 м, ширина 12,8 м

Длина прямоугольника равна 1.На 1 метр меньше ширины. Периметр прямоугольника 49,4 метра. Найдите размеры прямоугольника.

Периметр прямоугольника составляет 150 футов. Длина прямоугольника в два раза больше ширины. Найдите длину и ширину прямоугольника.

Длина прямоугольника в три раза больше ширины. Периметр прямоугольника 72 фута. Найдите длину и ширину прямоугольника.

Длина прямоугольника на три метра меньше двойной ширины.Периметр прямоугольника — 36 метров. Найдите размеры прямоугольника.

Длина прямоугольника на пять дюймов больше, чем в два раза ширины. По периметру 34 дюйма. Найдите длину и ширину.

Периметр прямоугольного поля 560 ярдов. Длина на 40 ярдов больше ширины. Найдите длину и ширину поля.

Периметр прямоугольного атриума составляет 160 футов. Длина на 16 футов больше ширины. Найдите длину и ширину атриума.

Прямоугольная парковка имеет периметр 250 футов. Длина на пять футов больше, чем в два раза ширины. Найдите длину и ширину парковки.

Прямоугольный коврик имеет периметр 240 дюймов. Длина на 12 дюймов больше, чем в два раза ширины. Найдите длину и ширину коврика.

Повседневная математика

Криста хочет поставить забор вокруг своей треугольной клумбы. Стороны клумбы шесть футов, восемь футов и 10 футов. Сколько футов ограды ей понадобится, чтобы ограждать клумбу?

Хосе только что убрал детский игровой набор со своего заднего двора, чтобы освободить место для прямоугольного сада.Он хочет поставить забор вокруг сада, чтобы не пускать собаку. У него в гараже есть 50-футовый рулон забора, который он планирует использовать. Чтобы поместиться на заднем дворе, ширина сада должна составлять 10 футов. Как долго он сможет сделать другую длину?

Письменные упражнения

Если вам нужно положить плитку на кухонный пол, вам нужно знать периметр или площадь кухни? Объясните свои рассуждения.

Если вам нужно поставить забор вокруг вашего заднего двора, вам нужно знать периметр или площадь заднего двора? Объясните свои рассуждения.

Посмотрите на две цифры ниже.

ⓐ Какая фигура имеет большую площадь?
ⓑ Какая из них, похоже, имеет больший периметр?
ⓒ Теперь вычислите площадь и периметр каждой фигуры.
ⓓ У кого площадь больше?
ⓔ У кого периметр больше?

ⓐ Ответы могут быть разными.
ⓑ Ответы будут разными.
ⓒ Ответы будут разными.
ⓓ Площади такие же.
ⓔ Прямоугольник 2 × 8 имеет больший периметр, чем квадрат 4 × 4.

Напишите задачу о геометрии, которая относится к вашему жизненному опыту, затем решите ее и объясните все свои шаги.

Самопроверка

ⓐ После выполнения упражнений используйте этот контрольный список, чтобы оценить свое мастерство в достижении целей этого раздела.

ⓑ Что этот контрольный список говорит вам о вашем мастерстве в этом разделе? Какие шаги вы предпримете для улучшения?

9.6: Использование свойств прямоугольников, треугольников и трапеций (часть 1)

В этом разделе мы продолжим работу с геометрическими приложениями.Мы добавим еще несколько свойств треугольников и узнаем о свойствах прямоугольников и трапеций.

Что такое линейные, квадратные и кубические меры

Когда вы измеряете свой рост или длину садового шланга, вы используете линейку или рулетку (рис. \ (\ PageIndex {1} \)). Рулетка может напоминать вам линию — вы используете ее для линейной меры , которая измеряет длину. Дюйм, фут, ярд, миля, сантиметр и метр — единицы линейного измерения.

Рисунок \ (\ PageIndex {1} \) — Эта рулетка измеряет дюймы сверху и сантиметры снизу.

Если вы хотите узнать, сколько плитки нужно для покрытия пола или размер стены, которую нужно покрасить, вам нужно знать площадь , меру области, необходимой для покрытия поверхности. Измеренная площадь составляет квадратных единиц . Мы часто используем квадратные дюймы, квадратные футы, квадратные сантиметры или квадратные мили для измерения площади. Квадратный сантиметр — это квадрат, каждая сторона которого равна одному сантиметру (см). Квадратный дюйм — это квадрат, каждая сторона которого составляет один дюйм (рисунок \ (\ PageIndex {2} \)).

Рисунок \ (\ PageIndex {2} \) — Квадратные меры имеют стороны, каждая из которых имеет длину 1 единицу.

На рисунке \ (\ PageIndex {3} \) показан прямоугольный коврик длиной 2 фута и шириной 3 фута. Каждый квадрат имеет ширину 1 фут и длину 1 фут, или 1 квадратный фут. Коврик состоит из 6 квадратов. Площадь коврика 6 квадратных футов.

Рисунок \ (\ PageIndex {3} \). Коврик состоит из шести квадратов по 1 квадратный фут каждый, поэтому общая площадь ковра составляет 6 квадратных футов.

Когда вы измеряете, сколько требуется для заполнения контейнера, например количество бензина, которое может поместиться в бак, или количество лекарства в шприце, вы измеряете объем . Объем измеряется в кубических единицах , например в кубических дюймах или кубических сантиметрах. При измерении объема прямоугольного твердого тела вы измеряете, сколько кубиков заполняет контейнер. Мы часто используем кубические сантиметры, кубические дюймы и кубические футы. Кубический сантиметр — это куб, имеющий размер по одному сантиметру с каждой стороны, а кубический дюйм — это куб, который измеряет один дюйм с каждой стороны (рисунок \ (\ PageIndex {4} \)).

Рисунок \ (\ PageIndex {4} \) — Кубические меры имеют стороны, равные 1 единице длины.

Предположим, что куб на рисунке \ (\ PageIndex {5} \) имеет размеры 3 дюйма с каждой стороны и разрезан по показанным линиям. Сколько в нем маленьких кубиков? Если бы мы разобрали большой куб на части, мы бы нашли 27 маленьких кубиков, каждый размером в один дюйм со всех сторон. Итак, каждый маленький куб имеет объем 1 кубический дюйм, а объем большого куба — 27 кубических дюймов.

Рисунок \ (\ PageIndex {5} \) — Куб размером 3 дюйма с каждой стороны состоит из 27 кубов размером 1 дюйм или 27 кубических дюймов.

Пример \ (\ PageIndex {1} \):

Для каждого предмета укажите, будете ли вы использовать линейную, квадратную или кубическую меру: (a) количество коврового покрытия, необходимое в комнате (b) длина удлинительного шнура (c) количество песка в песочнице (d) длина занавески стержень (е) количество муки в канистре (е) размер крыши собачьей будки.

Решение

(a) Вы измеряете, какую площадь покрывает ковер, то есть площадь. квадратный метр
(b) Вы измеряете длину удлинителя, то есть длину. линейная мера
(c) Вы измеряете объем песка. мера кубическая
(d) Вы измеряете длину карниза. линейная мера
(e) Вы измеряете объем муки. мера кубическая
(f) Вы измеряете площадь крыши. квадратный метр

Упражнение \ (\ PageIndex {1} \):

Определите, будете ли вы использовать линейную, квадратную или кубическую меру для каждого элемента.(a) количество краски в банке (b) высота дерева (c) пол вашей спальни (d) диаметр велосипедного колеса (e) размер куска дерна (f) количество воды в бассейне

Ответьте на

куб.

Ответ b

линейная

Ответ c

квадрат

Ответ d

линейная

Ответ e

кв.

Ответ f

куб.

Упражнение \ (\ PageIndex {2} \):

Определите, будете ли вы использовать линейную, квадратную или кубическую меру для каждого элемента.(а) объем упаковочной коробки (б) размер дворика (в) количество лекарства в шприце (г) длина отрезка пряжи (д) размер жилого участка (е) высота флагштока

Ответьте на

куб.

Ответ b

кв.

Ответ c

куб.

Ответ d

линейная

Ответ e

кв.

Ответ f

линейная

Многие геометрические приложения включают определение периметра или площади фигуры.В повседневной жизни также существует множество применений периметра и площади, поэтому важно убедиться, что вы понимаете, что каждое из них означает.

Представьте себе комнату, которой нужна новая напольная плитка. Плитки состоят из квадратов, каждая сторона которых составляет один фут — один квадратный фут. Сколько таких квадратов нужно, чтобы покрыть пол? Это площадь пола.

Затем подумайте о том, чтобы положить в комнату новый плинтус после того, как выложили плитку. Чтобы выяснить, сколько полосок понадобится, необходимо знать расстояние по комнате.Вы можете использовать рулетку, чтобы измерить количество футов в комнате. Это расстояние — периметр.

Определение: периметр и площадь

Периметр — это мера расстояния вокруг фигуры.

Площадь — это мера поверхности, покрытой фигурой.

На рисунке \ (\ PageIndex {6} \) показана квадратная плитка размером 1 дюйм с каждой стороны. Если бы муравей обошел край плитки, он прошел бы 4 дюйма. Это расстояние — периметр плитки.

Так как плитка представляет собой квадрат размером 1 дюйм с каждой стороны, ее площадь составляет один квадратный дюйм. Площадь фигуры измеряется путем определения количества квадратных единиц, покрывающих фигуру.

Рисунок \ (\ PageIndex {6} \) — периметр = 4 дюйма, площадь = 1 квадратный дюйм. Когда муравей полностью обходит плитку по ее краю, он отслеживает периметр плитки. Площадь плитки — 1 квадратный дюйм.

Пример \ (\ PageIndex {2} \):

Каждая из двух квадратных плиток составляет 1 квадратный дюйм.Две плитки показаны вместе. а) Каков периметр фигуры? б) Что это за область?

Решение

(a) Периметр — это расстояние вокруг фигуры. По периметру 6 дюймов.

(b) Площадь — это поверхность, покрытая фигурой. Плитка составляет 2 квадратных дюйма, поэтому площадь составляет 2 квадратных дюйма.

Упражнение \ (\ PageIndex {3} \):

Найдите периметр (a) и площадь (b) фигуры:

Ответьте на

8 дюймов

Ответ b
3 кв.в дюймах

Упражнение \ (\ PageIndex {4} \):

Найдите периметр (a) и площадь (b) фигуры:

Ответьте на

8 см

Ответ b
4 кв. Сантиметра

Используйте свойства прямоугольников

Прямоугольник имеет четыре стороны и четыре прямых угла. Противоположные стороны прямоугольника имеют одинаковую длину. Мы называем одну сторону прямоугольника длиной L, а соседнюю сторону — шириной W.См. Рисунок \ (\ PageIndex {7} \).

Рисунок \ (\ PageIndex {7} \) — Прямоугольник имеет четыре стороны и четыре прямых угла. Стороны помечены буквой L для длины и W.

Периметр P прямоугольника — это расстояние вокруг прямоугольника. Если вы начнете с одного угла и обойдете прямоугольник, вы пройдете единицы L + W + L + W, или две длины и две ширины. Тогда периметр

\ [\ begin {split} P = L + & W + L + W \\ & или \\ P = 2L & + 2W \ end {split} \]

А как насчет площади прямоугольника? Вспомните прямоугольный коврик из начала этого раздела.Его длина составляла 2 фута, ширина — 3 фута, а площадь — 6 квадратных футов. См. Рисунок \ (\ PageIndex {8} \). Поскольку A = 2 • 3, мы видим, что площадь A равна длине L, умноженной на ширину W, поэтому площадь прямоугольника равна A = L • W.

Рисунок \ (\ PageIndex {8} \) — Площадь этого прямоугольного коврика составляет 6 квадратных футов, его длина умножена на ширину.

Определение: свойства прямоугольников

  • Прямоугольники имеют четыре стороны и четыре прямых (90 °) угла.
  • Длины противоположных сторон равны.
  • Периметр P прямоугольника равен сумме удвоенной длины и удвоенной ширины. См. Рисунок 9.19. $$ P = 2L + 2W $$
  • Площадь A прямоугольника равна длине, умноженной на ширину. $$ A = L \ cdot W $$

Для удобства работы с примерами в этом разделе мы переформулируем здесь стратегию решения проблем для геометрических приложений.

КАК: ИСПОЛЬЗОВАТЬ СТРАТЕГИЮ РЕШЕНИЯ ПРОБЛЕМ ДЛЯ ГЕОМЕТРИЧЕСКИХ ПРИЛОЖЕНИЙ

Шаг 1. Прочтите задачу и убедитесь, что вы понимаете все слова и идеи.Нарисуйте фигуру и напишите на ней указанную информацию.

Шаг 2. Определите , что вы ищете.

Шаг 3. Назовите то, что вы ищете. Выберите переменную для представления этого количества.

Шаг 4. Преобразуйте в уравнение, написав соответствующую формулу или модель для ситуации. Подставьте в данную информацию.

Шаг 5. Решите уравнение, используя хорошие методы алгебры.

Шаг 6. Отметьте ответ в задаче и убедитесь, что он имеет смысл.

Шаг 7. Ответьте на вопрос полным предложением.

Пример \ (\ PageIndex {3} \):

Длина прямоугольника 32 метра, ширина 20 метров. Найдите (а) периметр и (б) площадь.

Решение

(а)

Шаг 1. Прочтите проблему. Нарисуйте фигуру и напишите на ней указанную информацию.
Шаг 2. Определите , что вы ищете. периметр прямоугольника
Шаг 3. Имя . Выберите переменную для ее представления. Пусть P = периметр
Шаг 4. Перевести . Напишите соответствующую формулу. Заменять.
Шаг 5. Решите уравнение. $$ \ begin {split} P & = 64 + 40 \\ P & = 104 \ end {split} $$
Шаг 6. Проверьте . $$ \ begin {split} P & \ stackrel {?} {=} 104 \\ 20 + 32 + 20 + 32 & \ stackrel {?} {=} 104 \\ 104 & = 104 \; \ checkmark \ end {split} $$
Шаг 7. Ответьте на вопрос. Периметр прямоугольника 104 метра.

(б)

Шаг 1. Прочтите проблему. Нарисуйте фигуру и напишите на ней указанную информацию.
Шаг 2. Определите , что вы ищете. площадь прямоугольника
Шаг 3. Имя . Выберите переменную для ее представления. Пусть A = площадь
Шаг 4. Перевести . Напишите соответствующую формулу.Заменять.
Шаг 5. Решите уравнение. $$ A = 640 $$
Шаг 6. Проверьте . $$ \ begin {split} A & \ stackrel {?} {=} 640 \\ 32 \ cdot 20 & \ stackrel {?} {=} 640 \\ 640 & = 640 \; \ checkmark \ end {split} $$
Шаг 7. Ответьте на вопрос. Площадь прямоугольника 60 квадратных метров.

Упражнение \ (\ PageIndex {5} \):

Длина прямоугольника составляет 120 ярдов, а ширина — 50 ярдов. Найдите (а) периметр и (б) площадь.

Ответьте на

340 ярдов

Ответ b
6000 кв. Ярдов

Упражнение \ (\ PageIndex {6} \):

Длина прямоугольника 62 фута, ширина 48 футов. Найдите (а) периметр и (б) площадь.

Ответьте на

220 футов

Ответ b
2976 кв. Футов

Пример \ (\ PageIndex {4} \):

Найдите длину прямоугольника с периметром 50 дюймов и шириной 10 дюймов.

Решение

Шаг 1. Прочтите проблему. Нарисуйте фигуру и напишите на ней указанную информацию.
Шаг 2. Определите то, что вы ищете. длина прямоугольника
Шаг 3. Имя . Выберите переменную для ее представления. Пусть L = длина
Шаг 4. Перевести . Напишите соответствующую формулу. Заменять.
Шаг 5. Решите уравнение. $$ \ begin {split} 50 \ textcolor {red} {- 20} & = 2L + 20 \ textcolor {red} {- 20} \\ 30 & = 2L \\ \ dfrac {30} {\ textcolor {красный } {2}} & = \ dfrac {2L} {\ textcolor {red} {2}} \\ 15 & = L \ end {split} $$
Шаг 6. Чек . $$ \ begin {split} P & \ stackrel {?} {=} 50 \\ 15 + 10 + 15 + 10 & \ stackrel {?} {=} 50 \\ 50 & = 50 \; \ checkmark \ end {split} $$
Шаг 7. Ответьте на вопрос. Длина 15 дюймов.

Упражнение \ (\ PageIndex {7} \):

Найдите длину прямоугольника с периметром 80 дюймов и шириной 25 дюймов.

Ответ

15 дюймов

Упражнение \ (\ PageIndex {8} \):

Найдите длину прямоугольника с периметром 30 ярдов и шириной 6 ярдов.

Ответ

9 ярдов

В следующем примере ширина определяется через длину. Мы будем ждать, чтобы нарисовать фигуру, пока мы не напишем выражение для ширины, чтобы мы могли пометить одну сторону этим выражением.

Пример \ (\ PageIndex {5} \):

Ширина прямоугольника на два дюйма меньше его длины. По периметру 52 дюйма. Найдите длину и ширину.

Решение

Шаг 1. Прочтите проблему.
Шаг 2. Определите , что вы ищете. длина и ширина прямоугольника
Шаг 3. Имя . Выберите переменную для ее представления. Теперь мы можем нарисовать фигуру, используя эти выражения для длины и ширины.

Поскольку ширина определяется длиной, мы полагаем L = длина.Ширина на два фута меньше длины, поэтому мы полагаем L — 2 = шириной.

Шаг 4. Перевести . Напишите соответствующую формулу. Формула периметра прямоугольника связывает всю информацию. Подставьте в данную информацию.
Шаг 5. Решите уравнение. $$ 52 = 2L + 2L — 4 $$
Объедините похожие термины. $$ 52 = 4L — 4 $$
Добавьте по 4 с каждой стороны. $$ 56 = 4л $$
Разделить на 4. $$ \ begin {split} \ dfrac {56} {4} & = \ dfrac {4L} {4} \\ 14 & = L \\ 14 & = L \ end {split} $$ Длина 14 дюймов .
Теперь нам нужно найти ширину. Ширина L — 2. $$ \ begin {split} & L — 2 \\ & \ textcolor {red} {14} — 2 \\ & 12 \ end {split} $$ Ширина 12 дюймов.
Шаг 6. Проверьте . Так как 14 + 12 + 14 + 12 = 52, это работает!
Шаг 7. Ответьте на вопрос. Длина 14 футов, ширина 12 футов.

Упражнение \ (\ PageIndex {9} \):

Ширина прямоугольника на семь метров меньше его длины. Периметр — 58 метров. Найдите длину и ширину.

Ответ

18 кв.м, 11 кв.м

Упражнение \ (\ PageIndex {10} \):

Длина прямоугольника на восемь футов больше ширины.Периметр — 60 футов. Найдите длину и ширину.

Ответ

11 футов 19 футов

Пример \ (\ PageIndex {6} \):

Длина прямоугольника на четыре сантиметра больше ширины в два раза. По периметру 32 сантиметра. Найдите длину и ширину.

Решение

Шаг 1. Прочтите проблему.
Шаг 2. Определите то, что вы ищете. длина и ширина
Шаг 3. Имя . Выберите переменную для ее представления.

пусть W = ширина

Длина в четыре раза больше ширины более чем в два раза.

2w + 4 = длина

Шаг 4. Перевести . Напишите соответствующую формулу и замените ее в приведенной информации.
Шаг 5. Решите уравнение. $$ \ begin {split} 32 & = 4w + 8 + 2w \\ 32 & = 6w + 8 \ 24 & = 6w \\ 4 & = w \ quad width \\ 2w & + 4 \ quad length \\ 2 (\ textcolor {red} {4}) & + 4 \\ 12 & \ quad \; длина\; является\; 12 \; см \ ldotp \ end {split} $$
Шаг 6. Проверьте . $$ \ begin {split} p & = 2L + 2W \\ 32 & \ stackrel {?} {=} 2 \ cdot 12 + 2 \ cdot 4 \\ 32 & = 32 \; \ checkmark \ end {split} $$
Шаг 7. Ответьте на вопрос. Длина 12 см, ширина 4 см.

Упражнение \ (\ PageIndex {11} \):

Длина прямоугольника в восемь раз больше ширины в два раза. Периметр — 64 фута. Найдите длину и ширину.

Ответ

8 футов 24 фута

Упражнение \ (\ PageIndex {12} \):

Ширина прямоугольника в шесть раз меньше его длины в два раза.По периметру 18 сантиметров. Найдите длину и ширину.

Ответ

5 см, 4 см

Пример \ (\ PageIndex {7} \):

Площадь прямоугольной комнаты составляет 168 квадратных футов. Длина 14 футов. Какая ширина?

Решение

Шаг 1. Прочтите проблему.
Шаг 2. Определите , что вы ищете. ширина прямоугольной комнаты
Шаг 3. Имя . Выберите переменную для ее представления. Пусть W = ширина
Шаг 4. Перевести . Напишите соответствующую формулу и замените ее в приведенной информации. $$ \ begin {split} A & = LW \\ 168 & = 14W \ end {split} $$
Шаг 5. Решите уравнение. $$ \ begin {split} \ dfrac {168} {14} & = \ dfrac {14W} {14} \\ 12 & = W \ end {split} $$
Шаг 6. Проверьте . $$ \ begin {split} A & = LW \\ 168 & \ stackrel {?} {=} 14 \ cdot 12 \\ 168 & = 168 \; \ checkmark \ end {split} $$
Шаг 7. Ответьте на вопрос. Ширина комнаты 12 футов.

Упражнение \ (\ PageIndex {13} \):

Площадь прямоугольника составляет 598 квадратных футов.Длина 23 фута. Какая ширина?

Ответ

26 футов

Упражнение \ (\ PageIndex {14} \):

Ширина прямоугольника 21 метр. Площадь 609 кв. Какая длина?

Ответ

29 кв.м

Пример \ (\ PageIndex {8} \):

Периметр прямоугольного бассейна составляет 150 футов. Длина на 15 футов больше ширины.Найдите длину и ширину.

Решение

Шаг 1. Прочтите проблему. Нарисуйте фигуру и напишите на ней указанную информацию.
Шаг 2. Определите , что вы ищете. длина и ширина бассейна
Шаг 3. Имя . Выберите переменную для ее представления. Длина на 15 футов больше ширины.

Пусть W = ширина

W + 15 = длина

Шаг 4. Перевести . Напишите соответствующую формулу и замените ее.
Шаг 5. Решите уравнение. $$ \ begin {split} 150 & = 2w + 30 + 2w \\ 150 & = 4w + 30 \\ 120 & = 4w \\ 30 & = w \ quad the \; ширина\; из\; \; бассейн \\ w & + 15 \ quad the \; длина\; из\; \; бассейн \\ \ textcolor {красный} {30} & + 15 \\ 45 & \ end {split} $$
Шаг 6. Чек . $$ \ begin {split} p & = 2L + 2W \\ 150 & \ stackrel {?} {=} 2 (45) + 2 (30) \\ 150 & = 150 \; \ checkmark \ end {split} $$
Шаг 7. Ответьте на вопрос. Длина бассейна 45 футов, ширина 30 футов.

Упражнение \ (\ PageIndex {15} \):

Периметр прямоугольного бассейна составляет 200 футов. Длина на 40 футов больше ширины.Найдите длину и ширину.

Ответ

30 футов 70 футов

Упражнение \ (\ PageIndex {16} \):

Длина прямоугольного сада на 30 ярдов больше ширины. Периметр 300 ярдов. Найдите длину и ширину.

Ответ

60 ярдов, 90 ярдов

4.7 Прикладные задачи оптимизации — Расчетный том 1

Шаг 1: Нарисуйте прямоугольную рамку и введите переменную xx, чтобы представить длину каждой стороны квадратного основания; пусть yy представляет высоту поля.Пусть SS обозначает площадь поверхности коробки с открытым верхом.

Рис. 4.69. Мы хотим минимизировать площадь квадратной коробки с заданным объемом.

Шаг 2: Нам нужно минимизировать площадь поверхности. Следовательно, нам нужно минимизировать S.S.

Шаг 3: Так как у коробки открытый верх, нам нужно только определить площадь четырех вертикальных сторон и основания. Площадь каждой из четырех вертикальных сторон равна x · y.x · y. Площадь основания x2.x2. Следовательно, площадь ящика

Шаг 4: Так как объем этого блока равен x2yx2y, а объем задан как 216in.3216 дюймов 3, уравнение связи —

Решая уравнение связи для y, y, получаем y = 216×2.y = 216×2. Следовательно, мы можем записать площадь поверхности как функцию только от xx:

S (х) = 4х (216х2) + х2. S (х) = 4х (216х2) + х2.

Следовательно, S (x) = 864x + x2. S (x) = 864x + x2.

Шаг 5: Поскольку мы требуем, чтобы x2y = 216, x2y = 216, мы не можем иметь x = 0.x = 0. Следовательно, нам нужно x> 0.x> 0. С другой стороны, xx может иметь любое положительное значение. Обратите внимание, что когда xx становится большим, высота прямоугольника yy становится соответственно маленькой, так что x2y = 216.х2у = 216. Точно так же, когда xx становится маленьким, высота коробки становится соответственно большой. Мы заключаем, что область представляет собой открытый неограниченный интервал (0, ∞). (0, ∞). Обратите внимание, что, в отличие от предыдущих примеров, мы не можем свести нашу проблему к поиску абсолютного максимума или абсолютного минимума на замкнутом ограниченном интервале. Однако на следующем шаге мы выясним, почему эта функция должна иметь абсолютный минимум на интервале (0, ∞). (0, ∞).

Шаг 6: Обратите внимание, что при x → 0 +, x → 0 +, S (x) → ∞, S (x) → ∞.Кроме того, при x → ∞, x → ∞, S (x) → ∞, S (x) → ∞. Поскольку SS — непрерывная функция, стремящаяся к бесконечности на концах, она должна иметь абсолютный минимум при некотором x∈ (0, ∞) .x∈ (0, ∞). Этот минимум должен быть в критической точке S.S. Производная —

.
S ′ (x) = — 864×2 + 2x. S ′ (x) = — 864×2 + 2x.

Следовательно, S ′ (x) = 0S ′ (x) = 0, когда 2x = 864×2.2x = 864×2. Решая это уравнение относительно x, x, мы получаем x3 = 432, x3 = 432, поэтому x = 4323 = 623.x = 4323 = 623. Поскольку это единственная критическая точка S, S, абсолютный минимум должен быть при x = 623x = 623 (см. Рисунок 4.70). Когда x = 623, x = 623, y = 216 (623) 2 = 323 дюйма, y = 216 (623) 2 = 323 дюйма. Следовательно, размеры коробки должны быть x = 623 дюйма x = 623 дюйма. и y = 323 дюйма, y = 323 дюйма. При таких размерах площадь поверхности составляет

м².
S (623) = 864623 + (623) 2 = 10843 дюйма. 2 S (623) = 864623 + (623) 2 = 10843 дюйма. 2

Рис. 4.70. Мы можем использовать график для определения размеров коробки с учетом объема и минимальной площади поверхности.

Rd Sharma 2020 for Class 6 Math Chapter 20

Страница № 20.10:
Вопрос 1:

Найдите периметры прямоугольников, длина и ширина которых указаны ниже:

(i) 7 см, 5 см
(ii) 5 см, 4 см
(iii) 7.5 см, 4,5 см

Ответ:

(i) Периметр прямоугольника = 2 × (длина + ширина)
∵ Длина = 7 см, ширина = 5 см
∴ Периметр = 2 × (7 + 5) = 2 × (12) = 24 см

( ii) Периметр прямоугольника = 2 × (длина + ширина)
∵ Длина = 5 см, ширина = 4 см
∴ Периметр = 2 × (5 + 4) = 2 × (9) = 18 см

(iii ) Периметр прямоугольника = 2 × (Длина + Ширина)
∵ Длина = 7.5 см, ширина = 4,5 см
∴ П риметр = 2 × (7,5 + 4,5) = 2 × (12) = 24 см

Страница № 20.10:
Вопрос 2:

Найдите периметры квадратов, стороны которых указаны ниже:

(i) 10 см
(ii) 5 м
(iii) 115,5 см

Ответ:

Периметр квадрата = 4 × (Длина одной стороны)

(i) Длина одной стороны = 10 см
Периметр = 4 × 10 = 40 см

(ii) Длина одной стороны = 5 м
Периметр = 4 × 5 = 20 м

(iii) Длина одной стороны = 115.5 см
Периметр = 4 × 115,5 = 462 см

Страница № 20.10:
Вопрос 3:

Найдите сторону квадрата, периметр которого равен:

(i) 16 м
(ii) 40 см
(iii) 22 см

Ответ:

Сторона квадрата = Периметр 4
(i) Периметр = 16 м
∴ Сторона этого квадрата = 16 4 = 4 м
(ii) Периметр = 40 см
∴ Сторона этого квадрата = 404 = 10 см
(iii ) Периметр = 22 см
∴ Сторона этого квадрата = 224 = 5.5см

Страница № 20.10:
Вопрос 4:

Найдите ширину прямоугольника с периметром 360 см и длиной

(i) 116 см
(ii) 140 см
(iii) 102 см

Ответ:

Периметр прямоугольника = 2 (длина + ширина)

∴ Ширина прямоугольника = периметр2- Длина

(i)
Периметр = 360 см
Длина = 116 см

∴ Ширина = 3602-116 = 180 -116 = 64 см

(ii)
Периметр = 360 см
Длина = 140 см

∴ Ширина = 3602-140 = 180-140 = 40 см
(iii)
Периметр = 360 см
Длина = 102 см

Ширина = 3602-102 = 180-102 = 78 см

Страница № 20.10:
Вопрос 5:

Прямоугольный кусок газона шириной 55 м и длиной 98 м. Найдите длину забора вокруг него.

Ответ:

Длина газона = 98 м
Ширина газона = 55 м
Длина ограждения вокруг лужайки = Периметр лужайки = 2 × (Длина + Ширина)
Периметр лужайки = 2 × (98 + 55) м = 2 × (153) = 306 м
Таким образом, длина забора вокруг газона = 306 м

Страница № 20.10:
Вопрос 6:

Сторона квадратного поля 65 м. Какой длины требуется ограждение вокруг него?

Ответ:

Сторона квадратного поля = 65 м
Длина ограждения вокруг квадратного поля = Периметр квадратного поля = 4 × (Сторона квадрата)
Периметр квадратного поля = 4 × 65 = 260 м
Таким образом, длина забора по квадратному полю = 260 м

Страница № 20.10:
Вопрос 7:

Две стороны треугольника равны 15 см и 20 см. Периметр треугольника — 50 см. Какая третья сторона?

Ответ:

Дано:
Периметр = 50 см
Длина первой стороны = 15 см
Длина второй стороны = 20 см
Нам нужно найти длину третьей стороны.
Периметр треугольника = Сумма всех трех сторон треугольника
∴ Длина третьей стороны = (Периметр треугольника) — (Сумма длин двух других сторон)
= 50 — (15 + 20)
= 50 — 35 = 15 см

Страница № 20.10:
Вопрос 8:

Трос длиной 20 м сложить в виде прямоугольника. Сколько прямоугольников можно образовать, сложив проволоку, если стороны положительные целые числа в метрах?

Ответ:

Предусмотрено, что трос длиной 20 м должен быть сложен в виде прямоугольника; следовательно, имеем:
Периметр прямоугольника = 20 м
⇒ 2 (Длина + Ширина) = 20 м
⇒ (Длина + Ширина) = 20/2 = 10 м

Так как длина и ширина являются целыми положительными числами в метрах. , следовательно, возможные размеры:
(1м, 9м), (2м, 8м), (3м, 7м), (4м, 6м) ​​и (5м, 5м)

Таким образом, можно сформировать пять прямоугольников с заданными провод.

Страница № 20.10:
Вопрос 9:

Квадратный участок земли, каждая сторона которого равна 100 м. Если для ограждения нужно использовать 3 слоя металлической проволоки, какая длина проволоки нужна?

Ответ:

Сторона квадратного поля = 100 м
Проволока, необходимая для ограждения квадратного поля = Периметр квадратного поля = 4 × Сторона квадратного поля
Периметр = 4 × 100 = 400 м
Этот периметр — длина провода, необходимого для забор в один слой.
Следовательно, длина проволоки, необходимая для трехслойного ограждения = 3 × 400 м = 1200 м

Страница № 20.10:
Вопрос 10:

Шиха проходит по квадрату со стороной 75 м. Прия проходит по прямоугольнику длиной 60 м и шириной 45 м. Кто преодолеет меньшее расстояние?

Ответ:

Шиха и Прия, бегая по квадратному и прямоугольному полю соответственно, фактически проходят расстояние, равное периметру этих полей.
∴ Расстояние, пройденное Шихой = Периметр квадрата = 4 × 75 м = 300 м
Аналогично, расстояние, пройденное Прией = Периметр прямоугольника = 2 × (60 + 45) = 2 × 105 = 210 м
Таким образом, Очевидно, что расстояние, пройденное Прией, меньше, чем пройденное Шикхой.

Страница № 20.10:
Вопрос 11:

Размеры фотографии 30 см × 20 см. Какая длина деревянной рамы нужна для обрамления картины?

Ответ:

Размеры фотографии = 30 см × 20 см
Итак, необходимая длина деревянной рамы = Периметр фотографии = 2 (Длина + Ширина)
= 2 × (30 + 20) см
= 2 × 50 см
= 100 см

Страница № 20.10:
Вопрос 12:

Длина прямоугольного поля 100 м. Если его периметр 300 м, какова его ширина?

Ответ:

Длина прямоугольного поля = 100 м
Периметр прямоугольного поля = 300 м
Периметр прямоугольника = 2 (Длина + Ширина)
Применяя вышеприведенную формулу, получаем:
Ширина прямоугольного поля = Периметр2-Длина = 3002-100 = 150-100 = 50 м

Страница № 20.10:
Вопрос 13:

Для крепления проволоки забора в саду длиной 70 м и шириной 50 м Арвинд купил металлические трубы для опор, через каждые 5 метров установил столб. Каждый столб был длиной 2 метра. Какова общая длина труб, которые он купил для столбов?

Ответ:

Длина сада = 70 м
Ширина сада = 50 м
Периметр сада = 2 × (Длина + Ширина)
= 2 × (70 + 50)
= 2 × 120 = 240 м
По периметру сада, считается, что Арвинд устанавливает столб через каждые 5 метров друг от друга.

Итак, необходимое количество стоек = 2405 = 48
∵ Длина каждой стойки = 2 м
∴ Общая необходимая длина трубы = 48 × 2 = 96 м

Страница № 20.11:
Вопрос 14:

Найдите стоимость ограждения прямоугольного парка длиной 175 м и шириной 125 м из расчета 12 рупий за метр.

Ответ:

Длина парка = 175 м
Ширина парка = 125 м
Периметр парка = 2 × (длина + ширина)
= 2 × (175 + 125)
= 2 × 300 = 600 м
Скорость ограждения = Рупий.12 за метр
Стоимость ограждения = рупий. 12 × 600 = рупий. 7 200

Страница № 20.11:
Вопрос 15:

Периметр правильного пятиугольника — 100 см. Какова длина каждой стороны?

Ответ:

Правильный пятиугольник — это замкнутый многоугольник, имеющий пять сторон равной длины.
Периметр правильного пятиугольника = 100 см
Периметр правильного пятиугольника = 5 × Сторона правильного пятиугольника
Следовательно, сторона правильного пятиугольника = Периметр5 = 1005 = 20 см

Страница № 20.11:
Вопрос 16:

Найдите периметр правильного шестиугольника, каждая сторона которого составляет 8 м.

Ответ:

Правильный шестиугольник — это замкнутый многоугольник, имеющий шесть сторон равной длины.
Сторона шестиугольника = 8 м
Периметр шестиугольника = 6 × Сторона шестиугольника
= 6 × 8 = 48 м

Страница № 20.11:
Вопрос 17:

Прямоугольный участок земли размером 0,7 км на 0,5 км. Каждая сторона должна быть ограждена четырьмя рядами проволоки. Какая длина провода нужна?

Ответ:

Размеры прямоугольного участка = 0,7 км × 0,5 км
Периметр прямоугольного участка = 2 (длина + ширина)
= 2 (0.7 + 0,5) км = 2 × 1,2 км = 2,4 км
Этот периметр равен одному ряду проволоки, необходимой для ограждения земли.
Следовательно, длина проволоки, необходимая для ограждения земли с помощью четырех рядов проволоки = 4 × 2,4 км
= 9,6 км

Страница № 20.11:
Вопрос 18:

Avneet покупает 9 квадратных тротуарных плиток со стороной 12 м каждая.Он укладывает их в виде квадрата.

(i) Каков периметр его расположения?
(ii) Шари не нравится его расположение. Она заставляет его разложить их, как крест. Каков периметр ее аранжировки?
(iii) У кого периметр больше?
(iv) Авнет задается вопросом, есть ли способ получить еще больший периметр. Вы можете найти способ сделать это? (Тротуарная плитка должна сходиться по полному краю, она не может быть сломана)

Ответ:

(i) Длина стороны одной плиты = 1/2 м.
При квадратном расположении одна сторона квадрата образована тремя плитами.
Итак, длина стороны квадрата = 3 × 12 = 3/2 м.
Периметр квадратного устройства = 4 × 32 = 6 м.
(ii) Поперечное расположение состоит из 8 сторон. Эти стороны образуют периферию конструкции и имеют размер 1 м каждая.
Кроме того, эта конструкция состоит из 4 других сторон размером 1/2 м каждая.
Итак, периметр креста расположения = (1 + 1/2 + 1 + 1 + 1/2 + 1 + 1 + 1/2 + 1 + 1 + 1/2 + 1)
= (8 + 2) = 10 м

(iii) Периметр поперечного расположения = 10 м
Периметр квадратного расположения = 6 м
Таким образом, периметр поперечного расположения больше периметра квадратного расположения.

(iv) Нет, эти плиты нельзя размещать по периметру более 10 м.

Страница № 20.14:
Вопрос 1:

Следующие рисунки нарисованы на бумаге в квадрате. Подсчитайте количество квадратов, окруженных каждой фигурой, и найдите ее площадь, приняв площадь каждого квадрата равной 1 см 2 . (Рис. 20.25).

Ответ:

(i) Имеется 16 полных квадратов данной формы.
∵ Площадь одного квадрата = 1 см 2
∴ Площадь этой формы = 16 × 1 = 16 см 2
(ii) Имеется 36 полных квадратов данной формы.
∵ Площадь одного квадрата = 1 см 2
∴ Площадь 36 квадратов = 36 × 1 = 36 см 2
(iii) Имеется 15 полных и 6 половинных квадратов данной формы.
∵ Площадь одного квадрата = 1 см 2
∴ Площадь этой формы = (15 + 6 × 12) = 18 см 2
(iv) Имеется 20 полных и 8 полуквадратных фигур данной формы.
∵ Площадь одного квадрата = 1 см 2
∴ Площадь этой формы = (20 + 8 × 12) = 24 см 2
(v) Есть 13 полных квадратов, 8 больше половины квадратов и 7 меньше половины квадратов заданной формы.
∵ Площадь одного квадрата = 1 см 2
∴ Площадь этой формы = (13 + 8 × 1) = 21 см 2

(vi) Есть 8 полных квадратов, 6 квадратов больше половины и 4 меньше половины квадратов заданной формы.
∵ Площадь одного квадрата = 1 см 2
∴ Площадь этой формы = (8 + 6 × 1) = 14 см 2

Страница № 20.14:
Вопрос 2:

На листе бумаги в квадрате нарисуйте (i) прямоугольник, (ii) треугольник, (iii) любую неправильную замкнутую фигуру. Найдите приблизительную площадь каждого квадрата, подсчитав количество полных квадратов, больше половины и ровно половину.

Ответ:

(i) Прямоугольник: содержит 18 полных квадратов.
Если предположить, что площадь одного полного квадрата равна 1 см 2 , то площадь этого прямоугольника будет 18 см 2 .

(ii) Треугольник: этот треугольник содержит 4 полных квадрата, 6 больше половины квадратов и 6 меньше половины квадратов.
Если мы предположим, что площадь одного полного квадрата равна 1 см 2 , тогда площадь этой формы = (4 + 6 × 1) = 10 см 2

(iii) Любая неправильная фигура: Эта фигура состоит из из 10 полных квадратов, 1 ровно половину квадрата, 7 больше половины квадратов и 6 меньше половины квадратов.
Если предположить, что площадь одного полного квадрата равна 1 см 2 , то площадь этой формы = (10 + 1 × 12 + 7 × 1) = 17.5 см 2

Страница № 20.14:
Вопрос 3:

Нарисуйте на миллиметровой бумаге любой круг. Подсчитайте квадраты и используйте их, чтобы оценить площадь круглой области.
Рисунок

Ответ:

Этот круг на квадратной бумаге состоит из 21 полных квадратов, 15 квадратов больше половины и 8 квадратов меньше половины.
Предположим, что площадь 1 квадрата равна 1 см 2 .
Если пренебречь меньше чем половиной квадратов и аппроксимировать больше половины квадрата как равное полному квадрату, мы получим:
Площадь этой формы = (21 + 15) = 36 см 2

Страница № 20.15:
Вопрос 4:

Используйте кальку и миллиметровую бумагу для сравнения площадей следующих пар фигур:

Ответ:

(i) (ii)
Используя кальку, мы обрисовали обе фигуры на миллиметровой бумаге.
(i) Этот рисунок содержит 4 полных квадрата, 9 больше половины квадратов и 9 меньше половины квадратов.
Предположим, что площадь одного квадрата равна 1 см 2 .
Если пренебречь квадратами меньше половины и считать площадь более половины квадратов равной площади полного квадрата, мы получим:
Площадь этой формы = (4 + 9) = 13 см 2
(ii) Эта фигура состоит из 8 полных квадратов, 11 больше половины квадратов и 10 меньше половины квадратов.
Предположим, что площадь одного квадрата равна 1 см 2 .
Если пренебречь квадратами меньше половины и считать площадь более половины квадратов равной площади полного квадрата, мы получим:
Площадь этой формы = (8 + 11) = 19 см 2

При сравнении площади этих двух форм, мы получаем, что площадь на рис. (ii) больше, чем на рис. (i).

Страница № 20.21:
Вопрос 1:

Найдите площадь прямоугольника, у которого

(i) Длина = 6 см, ширина = 3 см
(ii) Длина = 8 см, ширина = 3 см
(iii) Длина = 4.5 см, ширина = 2 см.

Ответ:

(i)
Площадь прямоугольника = Длина × Ширина
Длина = 6 см
Ширина = 3 см
∴ Площадь прямоугольника = 6 × 3 = 18 см 2
(ii)
Площадь прямоугольника = Длина × Ширина
Длина = 8 см
Ширина = 3 см
∴ Площадь прямоугольника = 8 × 3 = 24 см 2
(iii)
Площадь прямоугольника = Длина × Ширина
Длина = 4,5 см
Ширина = 2 см
∴ Площадь прямоугольника = 4.5 × 2 = 9 см 2

Страница № 20.21:
Вопрос 2:

Найдите площадь квадрата со стороной:

(i) 5 см
(ii) 4,1 см
(iii) 5,5 см
(iv) 2,6 см

Ответ:

Площадь квадрата = Сторона × Сторона
(i) Сторона квадрата = 5 см
Площадь квадрата = 5 × 5 = 25 см 2
(ii) Сторона квадрата = 4.1 см
Площадь квадрата = 4,1 × 4,1 = 16,81 см 2
(iii) Сторона квадрата = 5,5 см
Площадь квадрата = 5,5 × 5,5 = 30,25 см 2
(iv) Сторона квадрата = 2,6 см
Площадь квадрата = 2,6 × 2,6 = 6,76 см 2

Страница № 20.21:
Вопрос 3:

Площадь прямоугольника 49 см 2 , ширина 2.8 см, найдите длину прямоугольника.

Ответ:

Площадь = 49 см 2
Ширина = 2,8 см
Площадь прямоугольника = длина × ширина
∴ Длина = ширина области = 492,8 = 17,5 см

Страница № 20.21:
Вопрос 4:

Сторона квадрата 70 см. Найдите его площадь и периметр.

Ответ:

Сторона квадрата = 70 см
Площадь квадрата = Сторона × Сторона
= 70 × 70 = 4900 см 2
Периметр квадрата = 4 × Сторона = 4 × 70 = 280 см

Страница № 20.21:
Вопрос 5:

Площадь прямоугольника составляет 225 см 2 , а его одна сторона равна 25, найдите его другую сторону.

Ответ:

Площадь = 225 см 2
Одна из сторон = 25 см
Площадь прямоугольника = Произведение длин двух его сторон
Другая сторона = Площадь Сторона = 22525 = 9 см

Страница № 20.21:
Вопрос 6:

Что произойдет с площадью прямоугольника, если его

(i) Длина и ширина утроятся.
(ii) Длина увеличена вдвое, а ширина такая же.
(iii) Длина увеличена вдвое, а ширина уменьшена вдвое.

Ответ:

(i) Если длина и ширина прямоугольника утроены.
Пусть начальная длина и ширина равны l и b соответственно.
Исходная площадь = л × b = фунт
Теперь длина и ширина утроены, что означает, что они становятся в три раза больше их первоначального значения.
Следовательно,
Новая длина = 3 л
Новая ширина = 3b
Новая площадь = 3 л × 3 b = 9 фунтов
Таким образом, площадь прямоугольника станет в 9 раз больше его площади. оригинальная площадь.

(ii) Если длина увеличена вдвое, а ширина такая же.
Пусть начальная длина и ширина будут соответственно l и b .
Исходная площадь = l × b = lb
Теперь длина увеличена вдвое, а ширина осталась прежней.
Следовательно,
Новая длина = 2 l
Новая ширина = b
Новая площадь = 2l × b = 2 фунта
Таким образом, площадь прямоугольника станет в 2 раза больше его первоначальной площади.

(iii) Если длина увеличена вдвое, а ширина уменьшена вдвое.
Пусть начальная длина и ширина будут соответственно l и b .
Исходная площадь = l × b = lb
Теперь длина увеличена вдвое, а ширина уменьшена вдвое.
Следовательно,
Новая длина = 2 л
Новая ширина = b / 2
Новая площадь = 2l × b / 2 = фунт
Новая площадь также составляет фунтов Это означает, что области остаются неизменными. одно и тоже.

Страница № 20.21:
Вопрос 7:

Что произойдет с площадью квадрата, если его сторона равна

(i) утроится
(ii) увеличится вдвое.

Ответ:

(i) Пусть исходная сторона квадрата равна s .
Исходная площадь = с × с = с 2
Если сторона квадрата утроится, новая сторона будет равна 3 с .
Новая область = 3 с × 3 с = 9 с 2
Это означает, что площадь увеличивается в 9 раз по сравнению с исходной площадью.
(ii) Пусть исходная сторона квадрата равна s .
Исходная площадь = с × с = с 2
Если сторона квадрата увеличивается на половину, новая сторона = ( с +12 с) = 32 с
Новая область = 32 s × 32 s = 94 s
Это означает, что площадь становится в 94 раз больше, чем исходная площадь.

Страница № 20.21:
Вопрос 8:

Найдите периметр прямоугольника площадью 500 см 2 и шириной 20 см.

Ответ:

Площадь = 500 см 2
Ширина = 20 см
Площадь прямоугольника = Длина × Ширина
Следовательно,
Длина = Ширина области = 50020 = 25 см
Периметр прямоугольника = 2 (Длина + Ширина)
= 2 (25 + 20) см = 2 × 45 см = 90 см

Страница № 20.21:
Вопрос 9:

Прямоугольник имеет площадь, равную площади квадрата со стороной 80 см.Если ширина прямоугольника 20 см, найдите его длину.

Ответ:

Сторона квадрата = 80 см
Площадь квадрата = Сторона × Сторона = 80 × 80 = 6400 см 2
Учитывая, что:
Площадь прямоугольника = Площадь квадрата = 6400 см 2
Ширина прямоугольник = 20 см
Применяя формулу:
Длина прямоугольника = AreaBreadth
Получаем:
Длина прямоугольника = 640020 = 320 см

Страница № 20.21:
Вопрос 10:

Площадь прямоугольника шириной 17 см 340 см 2 . Найдите периметр прямоугольника.

Ответ:

Площадь прямоугольника = 340 см 2
Ширина прямоугольника = 17 см
Применяя формулу:
Длина прямоугольника = AreaBreadth
Получаем:
Длина прямоугольника = 34017 = 20 см
Периметр прямоугольника = 2 (длина + ширина)
= 2 (20 + 17)
= 2 × 37
= 74 см

Страница № 20.21:
Вопрос 11:

Размер мраморной плитки 15 см × 20 см. Сколько плитки потребуется, чтобы покрыть стену размером 4 м × 6 м? Также найдите общую стоимость плиток из расчета 2 рупия за плитку.

Ответ:

Размеры плитки = 15 см × 20 см
Размеры стены = 4 м × 6 м = 400 см × 600 см (Поскольку, 1 м = 100 см, значит, 4 м = 400 см и 6 м = 600 см )
Площадь плитки = 15 см × 20 см = 300 см 2
Площадь стены = 400 см × 600 см = 2,40,000 см 2

Количество плиток, необходимых для покрытия стены = Площадь стена Площадь одной плитки = 240000300 = 800 плиток

Страница № 20.21:
Вопрос 12:

Мраморная плитка размером 10 см × 12 см. Сколько плиток потребуется, чтобы покрыть стену размером
3 м × 4 м? Также найдите общую стоимость плиток из расчета 2 рупия за плитку.

Ответ:

Размер плитки = 10 см × 12 см
Размер стены = 3 м × 4 м = 300 см × 400 см (Поскольку, 1 м = 100 см, значит, 3 м = 300 см и 4 м = 400 см )
Площадь плитки = 10 см × 12 см = 120 см 2
Площадь стены = 300 см × 400 см = 1,20 000 см 2
Количество плиток, необходимых для покрытия стены = Площадь стены одной плитки = 120000120 = 1000 плиток
Стоимость плитки по курсу рупий.2 на плитку = 2 × 1000 = рупий. 2 000

Страница № 20.21:
Вопрос 13:

Одна сторона квадратного участка 250 м, найдите стоимость выравнивания из расчета 2 рупия за квадратный метр.

Ответ:

Сторона квадратного участка = 250 м
Площадь квадратного участка = Сторона × Сторона = 250 × 250 = 62 500 м 2
Скорость выравнивания участка = рупий.2 за м 2
Стоимость выравнивания квадратного участка = Rs. 62,500 × 2 = рупий. 1,25,000

Страница № 20.22:
Вопрос 14:

Следующие рисунки разбиты на прямоугольники. Найдите свои районы. (Размеры указаны в сантиметрах)

Ответ:

(i) Эта фигура состоит из двух прямоугольников II и IV и двух квадратов I и III.
Площадь квадрата I = Сторона × Сторона = 3 × 3 = 9 см 2
Аналогично, площадь прямоугольника II = (2 × 1) = 2 см 2
Площадь квадрата III = (3 × 3) = 9 см 2
площадь прямоугольника IV = (2 × 4) = 8 см 2
Таким образом, общая площадь этой фигуры = (Площадь квадрата I + Площадь прямоугольника II + Площадь квадрата III + Площадь прямоугольник IV) = 9 + 2 + 9 + 8 = 28 см 2

(ii) Эта фигура состоит из трех прямоугольников I, II и III.
Площадь прямоугольника I = длина × ширина = 3 × 1 = 3 см 2
Аналогично, площадь прямоугольника II = (3 × 1) = 3 см 2
площадь прямоугольника III = (3 × 1) = 3 см 2
Таким образом, общая площадь этой фигуры = (Площадь прямоугольника I + площадь прямоугольника II + площадь прямоугольника III) = 3 + 3 + 3 = 9 см 2

Страница № 20.22:
Вопрос 15:

Разделите следующие фигуры на прямоугольники и найдите площадь каждой.(Размеры указаны в сантиметрах)

Ответ:

(i) Этот рисунок состоит из двух прямоугольников I и II.
Площадь прямоугольника I = Длина × Ширина = 10 × 2 = 20 см 2
Аналогично, площадь прямоугольника II = 10 × 32 = 15 см 2
Таким образом, общая площадь этой фигуры = (Площадь прямоугольника I + Площадь прямоугольника II) = 20 + 15 = 35 см 2

(ii) Эта фигура состоит из двух квадратов I и III и одного прямоугольника II.
Площадь квадрата I = Площадь квадрата III = Сторона × Сторона = 7 × 7 = 49 см 2
Аналогично, площадь прямоугольника II = (21 × 7) = 147 см 2
Таким образом, общая площадь этого рисунок = (Площадь квадрата I + Площадь прямоугольника II + Площадь квадрата III) = 49 + 49 + 147 = 245 см 2

(iii) Эта фигура состоит из двух прямоугольников I и II.
Площадь прямоугольника I = Длина × Ширина = 5 × 1 = 5 см 2
Аналогично, площадь прямоугольника II = 4 × 1 = 4 см 2
Таким образом, общая площадь этой фигуры = (Площадь прямоугольника I + Площадь прямоугольника II) = 5 + 4 = 9 см 2

Страница № 20.22:
Вопрос 16:

Сколько плиток размером 5 см и 12 см потребуется для размещения области, длина и ширина которой составляют соответственно:

(i) 100 см и 144 см
(ii) 70 см и 36 см

Ответ:

(i) Размер плитки = 5 см × 12 см
Размер области = 100 см × 144 см
Площадь плитки = 5 см × 12 см = 60 см 2
Площадь области = 100 см × 144 см = 14 400 см 2
Количество плиток, необходимых для покрытия области = Площадь области Площадь одной плитки = 1440060 = 240 плиток
(ii) Размер плитки = 5 см × 12 см
Размер области = 70 см × 36 см
Площадь плитки = 5 см × 12 см = 60 см 2
Площадь области = 70 см × 36 см = 2,520 см 2
Количество плиток, необходимых для покрытия области = Площадь области Площадь одной плитки = 252060 = 42 плитки

Страница № 20.23:
Вопрос 1:

Стороны прямоугольника находятся в соотношении 5: 4. Если его периметр 72 см, то его длина равна

(a) 40 см
(b) 20 см
(c) 30 см
(d) 60 см

Ответ:

(b) 20 см
Пусть стороны прямоугольника равны 5 x и 4 x . (Так как они находятся в соотношении 5: 4)
Теперь периметр прямоугольника = 2 (длина + ширина)
72 = 2 (5 x + 4 x )
72 = 2 × 9 x
72 = 18 x
x = 7218 = 4
Таким образом, длина прямоугольника = 5 x = 5 × 4 = 20 см

Страница № 20.23:
Вопрос 2:

Стоимость ограждения прямоугольного поля длиной 34 м и шириной 18 м по 2,25 рупий за метр составляет

(a) 243
рупий (b) 234
рупий (c) 240
рупий (d) 334

рупий

Ответ:

(б) рупий. 234
Для ограждения прямоугольного поля нам нужно найти периметр прямоугольника.
Длина прямоугольника = 34 м
Ширина прямоугольника = 18 м
Периметр прямоугольника = 2 (длина + ширина)
= 2 (34 + 18) м
= 2 × 52 м = 104 м
Стоимость ограждения поле по ставке рупий.2,25 за метр = рупий. 104 × 2,25 = рупий. 234

Страница № 20.23:
Вопрос 3:

Если стоимость ограждения прямоугольного поля в рупиях. 7,50 за метр составляет рупий. 600, а длина поля 24 м, то ширина поля

(а) 8 м
(б) 18 м
(в) 24 м
(г) 16 м

Ответ:

(d) 16 м
Стоимость ограждения прямоугольного поля = Rs.600
Скорость ограждения поля = рупий. 7,50 за м
Следовательно, периметр поля = Стоимость ограждения Скорость ограждения = 6007,50 = 80 м
Теперь длина поля = 24 м
Следовательно, ширина поля = Периметр2- Длина
= 802-24 = 16 м

Страница № 20.23:
Вопрос 4:

Стоимость установки забора вокруг квадратного поля 2 рупия.50 за метр — 200 рупий. Длина каждой стороны поля составляет

(a) 80 м
(b) 40 м
(c) 20 м
(d) Ни одна из этих

Ответ:

(c) 20 м

Стоимость ограждения квадратного поля = Rs. 200

Ставка ограждения поля = руп. 2,50

Теперь периметр квадратного поля = Стоимость ограждения Скорость ограждения = 2002,50 = 80 м

Периметр квадрата = 4 × Сторона квадрата

Следовательно, сторона квадрата = Периметр4 = 804 = 20 м

Страница № 20.23:
Вопрос 5:

Длина прямоугольника в три раза больше его ширины. Если длина диагонали 810 м, то периметр прямоугольника равен

(а) 1510 м
(б) 1610 м
(в) 2410 м
(г) 64 м

Ответ:

(d) 64 м
Рассмотрим прямоугольник ABCD.
Также предположим, что ширина прямоугольника, т.е.э., ВС быть х м.
Дано, что длина в три раза больше ширины прямоугольника.
Следовательно, длина прямоугольника, т.е. AB = 3 x м

Итак, AC — это диагональ прямоугольника.
В прямоугольном треугольнике ABC.
AC 2 = AB 2 + BC 2 {Используя теорему Пифагора}
810 2 = 3×2 + x2
640 = 9 x 2 + x 2
640 = 10 x 2
x 2 = 64010 = 64
x = 64 = 8 м

Таким образом, ширина прямоугольника = x = 8 м
Аналогично, длина прямоугольника = 3 x = 3 × 8 = 24 м
Периметр прямоугольника = 2 (длина + ширина)
= 2 (24 + 8)
= 2 × 32 = 64 м

Страница № 20.23:
Вопрос 6:

Если диагональ прямоугольника в три раза больше его меньшей стороны, то его длина и ширина находятся в соотношении

(a) 3: 1
(b) 3: 1
(c) 2: 1
(d) 22: 1

Ответ:

(d) 22: 1
Предположим, что длина меньшей стороны прямоугольника, то есть BC, составляет x , а длина большей стороны , т.е.э., AB быть y .
Принято, что длина диагонали в три раза больше, чем меньшая сторона.
Следовательно, диагональ = 3 x = AC
Теперь, применяя теорему Пифагора, получаем:
(диагональ) 2 = (меньшая сторона) 2 + (большая сторона) 2

(AC) 2 = (AB) 2 + (BC) 2
(3 x ) 2 = ( x ) 2 + ( y ) 2
9 x 2 = x 2 + y 2
8 x 2 = y 2
Теперь, извлекая квадратные корни из обеих частей, получаем:
22 x = y
или, yx = 221

Таким образом, отношение большей стороны к меньшей стороне = 22: 1

Страница № 20.23:
Вопрос 7:

Соотношение площадей двух квадратов, диагональ одного из которых больше, чем у другого, составляет

(a) 1: 2
(b) 2: 3
(c) 3: 1
(d) 4: 1

Ответ:

(d) 4: 1

Пусть два квадрата — это ABCD и PQRS. Кроме того, диагональ квадрата PQRS вдвое больше диагонали квадрата ABCD.

PR = 2 AC

Теперь площадь квадрата = Diagonal22

Площадь PQRS = PR22

Аналогично, площадь ABCD = AC22

Согласно вопросу:

Если AC = x единиц, то , PR = 2 x ед.

Следовательно, Площадь PQRS Площадь ABCD = PR2 × 22 × AC2 = 2×2 × 22 × x2 = 41

Таким образом, соотношение площадей квадратов PQRS и ABCD = 4: 1

Страница № 20.23:
Вопрос 8:

Если соотношение площадей двух квадратов составляет 225: 256, то соотношение их периметров равно

(a) 225: 256
(b) 256: 225
(c) 15:16
(d) 16:15

Ответ:

(c) 15: 16

Пусть два квадрата будут ABCD и PQRS.

Далее, пусть длина каждой стороны ABCD и PQRS будет x и y, соответственно.

Следовательно

Площадь квадрата ABCDПлощадь квадрата PQRS = x2y2

⇒ x2y2 = 225256

Взяв квадратные корни с обеих сторон, получим:

xy = 1516

Теперь отношение их периметров:

Периметр кв. ABCDПриметр кв. PQRS = 4 × сторона кв. ABCD4 × Сторона кв. PQRS = 4x4y

⇒ Периметр кв. ABCDПриметр кв. PQRS = xy

⇒ Периметр кв. ABCDПриметр кв. PQRS = 1516

Таким образом, соотношение их периметров = 15: 16

Страница № 20.23:
Вопрос 9:

Если стороны квадрата делятся пополам, его площадь

(a) остается прежней
(b) становится половиной
(c) становится одной четвертой
(d) становится двойной

Ответ:

(c) становится одной четвертой

Пусть сторона квадрата равна x .

Тогда площадь = (Сторона × Сторона) = ( x × x ) = x 2

Если стороны делятся пополам, новая сторона = x2

Теперь новая область = x22 = x24

Хорошо видно, что площадь стала на четверть от своего предыдущего значения.

Страница № 20.23:
Вопрос 10:

Прямоугольный ковер имеет площадь 120 м 2 и периметр 46 метров. Длина его диагонали

(а) 15 м
(б) 16 м
(в) 17 м
(г) 20 м

Ответ:

(c) 17 м
Площадь прямоугольника = 120 м 2
Периметр = 46 м
Пусть стороны прямоугольника равны l и b .
Следовательно,
Площадь = фунтов = 120 м 2 … (1)
Периметр = 2 ( л + b ) = 46
или, ( л + b ) = 462 = 23 м … (2)
Теперь длина диагонали прямоугольника = l2 + b2
Итак, сначала находим значение ( l 2 + b 2 )
Используя тождество :
( л 2 + b 2 ) = ( л + b ) 2 — 2 ( фунтов ) [Из (1) и (2)]
Следовательно
( l 2 + b 2 ) = (23) 2 — 2 (120)
= 529 — 240 = 289
Таким образом, длина диагонали прямоугольника = l2 + b2 = 289 = 17 м

Страница № 20.23:
Вопрос 11:

Если соотношение между длиной и периметром прямоугольного участка составляет 1: 3, то соотношение между длиной и шириной участка составляет

(a) 1: 2
(b) 2: 1
(c) 3: 2
(г) 2: 3

Ответ:

(b) 2: 1
Дано, что Длина прямоугольника Периметр прямоугольника = 13
⇒ l2l + 2b = 13
После перекрестного умножения получаем:
3 l = 2l + 2b
l = 2b
⇒ lb = 21
Таким образом, соотношение длины и ширины составляет 2: 1.

Страница № 20.23:
Вопрос 12:

Если длина диагонали квадрата 20 см, то его периметр равен

(а) 102см
(б) 40 см
(в) 402см
(г) 200 см

Ответ:

(c) 40 2 см

Длина диагонали = 20 см

Длина стороны квадрата = Длина диагонали2 = 202 = 2 × 102
= 2 × 2 × 102 = 102 см

Следовательно, периметр квадрата квадрат = 4 × Сторона = 4 × 102 = 402 см

Страница № 20.24:
Вопрос 1:

Отметьте правильный вариант в следующем вопросе:

Если периметр квадрата 40 см, то длина каждой его стороны будет

(a) 20 см (b) 10 см (c) 5 см (d) 40 см

Ответ:

Длина каждой стороны квадрата = Периметр квадрата4 = 404 = 10 см

Следовательно, правильной альтернативой является вариант (b).

Страница № 20.24:
Вопрос 2:

Отметьте правильный вариант в следующем вопросе:

Площадь квадрата со стороной 14 см составляет

(a) 49 см 2 (b) 156 см 2 (c) 56 см 2 (d ) 196 см 2

Ответ:

Площадь квадрата = Сторона × Сторона
= 14 × 14
= 196 см 2

Следовательно, правильной альтернативой является вариант (d).

Страница № 20.24:
Вопрос 3:

Отметьте правильный вариант в следующем вопросе:

Длина диагонали квадрата составляет 20 см. Его площадь

(а) 400 см 2 (б) 200 см 2 (в) 300 см 2 (г) 1002 см 2

Ответ:

Площадь квадрата = 12 × диагональ × диагональ = 12 × 20 × 20 = 4002 = 200 см2

Следовательно, правильной альтернативой является вариант (б).

Страница № 20.24:
Вопрос 4:

Отметьте правильный вариант в следующем вопросе:

Длина и ширина прямоугольника области A увеличиваются вдвое. Площадь нового прямоугольника

(a) 2 A (b) A 2 (c) 4 A (d) Ни один из этих

Ответ:

Пусть длина и ширина данного прямоугольника равны l и b соответственно.

У нас есть,
A = фунтов ….. (i)

Кроме того,
длина нового прямоугольника, l ‘= 2 l и
ширина нового прямоугольника , b ‘= 2 b

Теперь площадь нового прямоугольника = l ‘ × b
= (2 l ) × (2 b )
= 4 фунтов
= 4 A [Использование (i)]

Следовательно, правильной альтернативой является вариант (c).

Страница № 20.24:
Вопрос 5:

Отметьте правильный вариант в следующем вопросе:

Максимальная длина стороны квадратного листа, которую можно отрезать от прямоугольного листа размером 8 м × 3 м, составляет

(a) 3 м (b) 4 м (в) 6 м (г) 4 м

Ответ:

Максимальная длина стороны квадратного листа, которую можно отрезать от прямоугольного листа размером 8 м × 3 м, составляет 3 м.

Следовательно, правильной альтернативой является вариант (а).

Страница № 20.24:
Вопрос 6:

Отметьте правильный вариант в следующем вопросе:

Периметр квадрата площадью 225 м 2 равен

(a) 15 м (b) 60 м (c) 225 м (d) 30 м

Ответ:

У нас есть, Площадь квадрата = 225 м2 Как, сторона квадрата = Площадь = 225 = 15 м, Итак, периметр квадрата = 4 × сторона = 4 × 15 = 60 м

Следовательно, правильная альтернатива — это вариант (б).

Страница № 20.24:
Вопрос 7:

Отметьте правильный вариант в следующем вопросе:

Площадь прямоугольника составляет 650 см 2 , а его ширина — 13 см. Периметр прямоугольника

(а) 63 см (б) 130 см (в) 100 см (г) 126 см

Ответ:

У нас есть, Площадь прямоугольника = 650 см2 и Ширина прямоугольника = 13 смAs, длина прямоугольника = AreaBreadth = 65013 = 50 см Таким образом, периметр прямоугольника = 2 × длина + ширина = 2 × 13 + 50 = 2. × 63 = 126 см

Следовательно, правильной альтернативой является вариант (d).

Страница № 20.24:
Вопрос 8:

Отметьте правильный вариант в следующем вопросе:

Сколько конвертов можно сделать из листа бумаги 72 на 48 см, если для каждого конверта требуется бумага размером 18 на 12 см?

(а) 4 (б) 8 (в) 12 (г) 16

Ответ:

Имеем, длина листа бумаги = 72 см, ширина листа бумаги = 48 см, длина конверта = 18 см и ширина конверта = 12 см. Площадь листа бумаги = длина × ширина = 72 × 48 см2 И площадь конверта = длина × ширина = 18 × 12 см2 Теперь количество конвертов, которое можно сделать, = Площадь листа бумаги Площадь конверта = 72 × 4818 × 12 = 4 × 4 = 16

Следовательно, правильной альтернативой является вариант (d).

Страница № 20.24:
Вопрос 9:

Отметьте правильный вариант в следующем вопросе:

Если диагональ квадрата 8 метров, то его площадь будет

(a) 8 м 2 (b) 4 м 2 (c) 16 м 2 (г) 6 м 2

Ответ:

Имеем, длина диагонали квадрата = 8 м, теперь площадь квадрата = 12 × диагональ × диагональ = 12 × 8 × 8 = 82 = 4 м2

Следовательно, правильной альтернативой является вариант (б).

Страница № 20.24:
Вопрос 10:

Отметьте правильный вариант в следующем вопросе:

Площадь заштрихованного пути на следующем рисунке равна

(a) 16 м 2 (b) 18 м 2 (c) 14 м 2 (г) 20 м 2

Ответ:

Площадь заштрихованной области = Площадь прямоугольника + Площадь равнобедренного прямоугольного треугольника l = длина × ширина + 12 × основание × высота = 8 × 2 + 12 × 2 × 2 = 16 + 2 = 18 м2

Следовательно, правильной альтернативой является вариант (б).

Страница № 20.24:
Вопрос 11:

Площадь прямоугольной площадки — 120 м. 2 , длина — 12 м. Найдите стоимость ограждения земли из расчета 125 фунтов стерлингов за метр.

Ответ:

У нас есть, Площадь прямоугольной земли = 120 м2 и Длина земли = 12 м Теперь, Ширина земли = Площадь земли Длина земли = 12012 = 10 мА Также, Периметр земли = 2 × Длина + Ширина = 2 × 12 + 10 = 2 × 22 = 44 м. Так как ставка ограждения земли = 125 per за метр, то стоимость ограждения земли = 125 × 44 = 5500

.

Страница № 20.24:
Вопрос 12:

Найдите периметр следующей фигуры:

Ответ:

Периметр фигуры = AB + BC + CD + DE + EF + FG + GH + HA = 10 + 10 + 20 + 30 + 20 + 20 + 10 + 20 = 140 м

Страница № 20.25:
Вопрос 13:

Комната длиной 8 м и шириной 6 м.Пол выложить прямоугольной плиткой размером 25 на 20 см. Сколько потребуется плитки? Найдите стоимость этих плиток по 25 фунтов за плитку.

Ответ:

Имеем,
Длина комнаты = 8 м = 800 см,
Ширина комнаты = 6 м = 600 см,
Длина плитки = 25 см и
Ширина плитки = 20 см

Теперь количество необходимых плиток = Площадь пола в комнате Площадь плитки = Длина комнаты × Ширина комнаты Длина плитки × Ширина плитки = 800 × 60025 × 20 = 960 Итак, количество плиток, необходимое для покрытия Этаж комнаты 960.Кроме того, поскольку стоимость 1 плитки = ₹ 25So, стоимость 960 таких плиток = 25 × 960 = ₹ 24000.

Страница № 20.25:
Вопрос 14:

Длина и ширина прямоугольного парка в соотношении 5: 3, а его периметр равен 128 м. Найдите территорию парка.

Ответ:

Пусть длина парка 5x м, а ширина 3x м.Так, периметр парка = 2 × Длина парка + Ширина парка = 2 × 5x + 3x = 2 × 8x = 16x м Но периметр парка = 128 м⇒16x = 128⇒x = 12816⇒x = 8∴ Длина парка = 5x = 5 × 8 = 40 м и ширина парка = 3x = 3 × 8 = 24 м Теперь площадь парка = Длина × Ширина = 40 × 24 = 960 м2

Итак, Площадь парка 960 м 2 2 .

Страница № 20.25:
Вопрос 15:

Общая стоимость покрытия комнаты из расчета 85 ₹ за м 2 составляет 5100.Если длина комнаты 8 метров, то найдите ее ширину.

Ответ:

As, общая стоимость настила = 5100 and и ставка настила = 85 ₹ за м2 Таким образом, площадь пола = Общая стоимость настила Стоимость напольного покрытия = 510085 = 60 м2 Также длина комнаты = 8 м Теперь, ширина комнаты = Площадь этажа Длина комнаты = 608 = 152 = 7,5 м

Итак, ширина комнаты 7,5 м.

Страница № 20.25:
Вопрос 16:

Квадратный лист стороной 5 см вырезается из прямоугольного куска алюминиевого листа длиной 9 см и шириной 6 см. Какая площадь осталась у алюминиевого листа?

Ответ:

У нас есть,
Сторона вырезанного квадратного листа = 5 см,
Длина прямоугольного листа = 9 см и
Ширина прямоугольного листа = 6 см

Теперь площадь вырезанного квадратного листа = Сторона × Сторона = 5 × 5 = 25 см2 Также площадь прямоугольного листа = Длина × Ширина = 9 × 6 = 54 см2 Таким образом, площадь оставшегося листа = Площадь прямоугольного листа — Площадь вырезанного квадратного листа = 54 -25 = 29 см2

Следовательно, площадь оставшегося алюминиевого листа составляет 29 см 2.

Страница № 20.25:
Вопрос 17:

Найдите общую стоимость выравнивания заштрихованной дорожки одинаковой ширины 2 метра, проложенной в прямоугольном поле, показанном ниже, если ставка за м 2 составляет 100.

Ответ:

Заштрихованный путь можно сформировать как два прямоугольника, как показано ниже:

Теперь площадь заштрихованного пути = 38 × 2 + 50 × 2 = 76 + 100 = 176 м2As, скорость выравнивания заштрихованного пути = ₹ 100 за м2 Итак, общая стоимость выравнивания заштрихованного пути = 176 × 100 = 17 600 ₹

Заявление об ограничении ответственности: ответ, данный в учебнике, неверен.То же было исправлено выше.

Страница № 20.25:
Вопрос 18:

Квадратный лист 2 см вырезается из каждого угла прямоугольного куска алюминиевого листа длиной 12 см и шириной 8 см. Найдите область оставленного алюминиевого листа.

Ответ:

У нас
Сторона каждого квадратного листа, вырезанная из каждого угла = 2 см,
Длина прямоугольного куска листа = 12 см и
Ширина прямоугольного куска листа = 8 см

As, площадь каждого квадрата = Сторона × Сторона = 2 × 2 = 4 см2 Таким образом, площадь четырех квадратных листов = 4 × 4 = 16 см2 Также площадь прямоугольного листа = Длина × Ширина = 12 × 8 = 96 см2 Теперь площадь листа осталась over = Площадь прямоугольного листа — Площадь квадратного листа = 96-16 = 80 см2

Таким образом, площадь оставшегося алюминиевого листа составляет 80 см 2 .

Страница № 20.25:
Вопрос 19:

Помещение длиной 12,5 м и шириной 8 м. На его пол постелен квадратный ковер со стороной 8 м. Найдите площадь пола, на которой нет коврового покрытия.

Ответ:

Имеем,
Длина комнаты = 12,5 м,
Ширина комнаты = 8 м и
Сторона квадратного ковра = 8 м

Площадь комнаты = Длина × Ширина = 12.5 × 8 = 100 м2 Также площадь квадратного ковра = Сторона × Сторона = 8 × 8 = 64 м2 Теперь площадь пола без ковра = Площадь комнаты — Площадь ковра = 100-64 = 36 кв.м

Страница № 20.25:
Вопрос 20:

Площадь квадратного поля 36 м 2 . Вокруг и снаружи прокладывается дорожка одинаковой ширины. Если площадь пути 28 м 2 , то найдите ширину пути.

Ответ:

As, площадь квадратного поля = 36 м2 Итак, сторона квадратного поля = 36 = 6 мА, а также площадь внешнего квадрата = Площадь квадрата + Площадь пути = 36 + 28 = 64 м2. , Сторона внешнего квадрата = 64 = 8 м Теперь ширина дорожки = Сторона внешнего квадрата-Сторона внутреннего квадрата 2 = 8-62 = 22 = 1 м

Итак, ширина дорожки равна 1 м.

Страница № 20.25:
Вопрос 21:

Периметр квадрата 16 см, значит его площадь __________ см 2 .

Ответ:

As, периметр квадрата = 16 см. Итак, сторона квадрата = Perimeter4 = 164 = 4 см. Теперь площадь квадрата = Сторона × Сторона = 4 × 4 = 16 см2.

Следовательно, площадь квадрата 16 см 2 .

Страница № 20.25:
Вопрос 22:

Длина и ширина прямоугольника 12 см и 5 см соответственно, тогда его диагональ __________ см.

Ответ:

Имеем, Длина прямоугольника = 12 см и Ширина прямоугольника = 5 см Теперь диагональ прямоугольника = Длина 3 + Ширина 3 = 122 + 52 = 144 + 25 = 169 = 13 см

Следовательно, диагональ прямоугольника 13 см.

Страница № 20.25:
Вопрос 23:

Периметр квадрата 225 м 2 2 _________.

Ответ:

As, площадь квадрата = 225 м2 Итак, сторона квадрата = Площадь = 225 = 15 м Теперь периметр квадрата = 4 × Сторона = 4 × 15 = 60 м

Следовательно, периметр квадрата 60 м .

Страница № 20.25:
Вопрос 24:

Площадь прямоугольника 120 см. 2 , Если ширина 6 см, то его длина _________.

Ответ:

У нас площадь прямоугольника = 120 см2 и ширина прямоугольника = 6 см, длина прямоугольника = AreaBreadth = 1206 = 20 см

Следовательно, длина прямоугольника 20 см .

Страница № 20.25:
Вопрос 25:

Если соотношение между длиной и периметром прямоугольного участка составляет 1: 3, то соотношение между длиной и шириной участка составляет _______.

Ответ:

Пусть длина прямоугольного участка равна x, а его периметр равен 3x. Поскольку ширина прямоугольного участка = Perimeter2-Length = 3×2-x = 3x-2×2 = x2 Теперь соотношение между длиной и шириной участка = LengthBreadth = xx2 = 2xx = 21 = 2: 1

Следовательно, соотношение между длиной и шириной прямоугольного участка составляет 2: 1 .

Страница № 20.5:
Вопрос 1:

Какие из следующих замкнутых кривых? Какие из них простые?

Ответ:

Рис. (Ii), (iii), (iv), (vi) и (vii) — замкнутые кривые, тогда как на рис. (ii), (iii), (iv) и (vi) — простые замкнутые кривые.

Страница № 20.5:
Вопрос 2:

Определите периметр замкнутой фигуры.

Ответ:

Длина границы замкнутой фигуры называется ее периметром.

Страница № 20.5:
Вопрос 3:

Найдите периметр каждой из следующих фигур:

Ответ:

Периметр = Сумма длин всех сторон замкнутой фигуры
(i)

Периметр = (4 + 2 + 1 + 5) см = 12 см

(ii)

Периметр = (23 + 35 + 40 + 35) см = 133 см

(iii)

Периметр = (15 + 15 + 15 + 15) см = 60 см

(iv)

Периметр = (3 + 3 + 3 + 3 + 3) см = 15 см

Просмотреть решения NCERT для всех глав класса 6

Калькулятор асфальтоукладчика

Обязательно используйте этот калькулятор асфальтоукладчика, прежде чем начинать работу над проектом внутреннего дворика! На вопрос «сколько брусчатки мне нужно?» когда-либо приходило вам в голову, этот инструмент окажет вам неоценимую помощь.Указав стоимость брусчатки, вы сможете рассчитать общую стоимость вашего патио. Калькулятор укладчика патио также позволит вам рассчитать идеальное количество материалов для вашего проекта и, как следствие, сэкономит ваши деньги, так как ни один асфальтоукладчик не пропадет даром.

Как пользоваться калькулятором асфальтоукладчика?

Калькулятор укладчика патио очень прост. Вот краткий набор инструкций:

1. Определите площадь вашего патио .Начните с выбора, является ли ваш внутренний дворик одиночным прямоугольником или имеет другую форму, и в этом случае вам нужно разделить его на идентичные прямоугольники , чтобы упростить расчет.

  • ПРИМЕЧАНИЕ : Не беспокойтесь, если форму вашего патио невозможно разделить на прямоугольники одинакового размера. Если размеры его подобластей различаются, просто вычисляет материал в каждой из них , выбирая вариант одного прямоугольника, а затем суммирует результаты .

  • Для одиночного прямоугольника введите длину и ширину внутреннего дворика, и вы получите его площадь .

  • Для нескольких идентичных прямоугольников добавьте номер подобластей , и калькулятор покажет вам общую площадь внутреннего дворика.

2. Теперь нужно определить размер кирпича для брусчатки размером . Обеспечьте одинарного кирпича шириной и длиной одинарного кирпича , чтобы получить площадь, которую он покрывает.В следующем разделе этой статьи вы найдете таблицу самых распространенных размеров брусчатки.

3. Когда все эти значения готовы, калькулятор покажет вам , сколько кирпичей необходимо для покрытия одного квадратного фута , а также общего количества брусчатки , которая вам понадобится для вашего проекта.

4. Осталось только рассчитать затраты:

  • Чтобы оценить общую стоимость брусчатки в размере , просто укажите цену одного кирпича , и калькулятор брусчатки сделает всю работу за вас.

  • Чтобы получить ориентировочную стоимость установки , положите стоимости установки на квадратный фут . Как правило, это стоит от 8 до 20 долларов, но цены будут варьироваться в зависимости от вашего местоположения.

  • Калькулятор стоимости брусчатки суммирует два вышеуказанных значения и покажет вам общих предполагаемых расходов проекта, как по стоимости брусчатки, так и по их установке профессионалами.

Обычные размеры асфальтоукладчика

Чаще всего используется размер брусчатки 4 «x 8» (10 см x 20 см в метрической системе).Однако и другие размеры вовсе не редкость.

Вот таблица наиболее широко используемых размеров брусчатки с соответствующими размерами в метрической системе:

Размер (дюймы) Размер (см) Площадь (фут²) Площадь (см²)
4 х 8 10 х 20 0,22 200
6 x 6 15 х 15 0.25 225
6 x 9 15 х 23 0,38 245
8 x 8 20 х 20 0,44 400
12 х 12 30 х 30 1,00 900
14 х 14 36 х 36 1,36 1296
12 х 18 30 х 46 1.50 1380

Если вам нужно рассчитать объем кирпича, толщина обычно составляет 2 дюйма (5 см). Единственным исключением являются кирпичи размером 14 дюймов на 14 дюймов и 12 дюймов на 18 дюймов. обычно толщина составляет 3 дюйма (7,6 см) и 1,5 дюйма (4 см) соответственно.

Использование калькулятора асфальтоукладчика — пример

Давайте вместе рассмотрим пример использования калькулятора дорожной плитки для террасы.

Джон решил выложить патио кирпичом. Естественно, его первой мыслью было: «Сколько брусчатки мне нужно?» Его расчеты были следующими:

1. Джон измерил общую площадь внутреннего дворика. Внутренний дворик Джона имеет форму, напоминающую четырехрукую звезду, поэтому он может разделить его на пять квадратов 15×15 футов . Чтобы посчитать это, он использует следующее уравнение:

  • площадь патио = ширина подобласти * длина подобласти * количество подобластей

  • Итак, в нашем примере:

    площадь патио = 15 футов * 15 футов * 5 = 1125 футов²

2. Затем Джон должен вычислить площадь, которую покрывает один дорожный дворик, и количество брусчатки на квадратный фут .Он выбрал брусчатку размером 6 на 6 дюймов.

  • После умножения длины кирпича на его ширину Джон знает, что площадь одного брусчатки составляет 36 кв. Дюймов.

  • Чтобы рассчитать, сколько брусчатки умещается на одном квадратном футе, необходима следующая формула:

    брусчатки на квадратный фут = 144 / площадь одинарного кирпича

  • Причина, по которой числитель равен 144, состоит в том, что в одном квадратном футе 144 квадратных дюйма. В примере Джона:

    брусчатки на квадратный фут = 144/36 = 4

3. Осталось посчитать общего количества асфальтоукладчиков, необходимых . Для этого Джон использует это уравнение:

  • общее количество брусчатки = брусчатка на квадратный фут * общая площадь проекта (округлено в большую сторону — если вы не получили полное число, вам может потребоваться округлить его в большую сторону и отрегулировать лишний кирпич по краям проекта)

  • Итак, фактический расчет выглядит следующим образом:

    общее количество брусчатки = 4 * 1,125 = 4500

4. Джон также хочет оценить общую стоимость брусчатки в . Он хочет купить брусчатку за 0,50 доллара.

  • стоимость брусчатки = 4500 * 0,50 доллара = 2250 долларов

  • Он также хотел бы, чтобы асфальтоукладчики установил профессионал. В его районе стоимость такой услуги составляет 9 долларов за квадратный фут, то есть:

    Стоимость установки = площадь патио * стоимость квадратного метра = 1 125 * 9 долларов США = 10 125 долларов США

Примечание по использованию калькулятора асфальтоукладчика

Как и во всех других строительных работах, укладка брусчатки подвержена ошибкам.Чтобы не тратить бензин, возвращаясь в магазин за дополнительными материалами, подумайте о том, чтобы покупать как минимум на 10% больше всего.

Если вам нужна помощь в подсчете нужного количества дополнительных материалов, попробуйте воспользоваться нашим процентным калькулятором, чтобы избавиться от любых сомнений.

Брусчатка для патио — не единственное, что вам нужно купить для своего проекта. Не забудьте принять во внимание песок и гравий, которые являются очень важными материалами при мощении патио.

А как насчет узоров?

Подсчитать, сколько брусчатки вам нужно, кажется простым и приятным, когда мы говорим о простых прямоугольных кирпичах, уложенных рядом друг с другом.Но вам может быть интересно, а что насчет шаблонов? Сколько брусчатки вам тогда нужно? Не волнуйтесь — решение этой проблемы тоже довольно простое.

Уловка состоит в том, чтобы собрать одну повторяющуюся часть узора и вычислить ее площадь, а затем умножить на количество повторов узора.

Например:

В случае этого конкретного шаблона, чтобы вычислить площадь, вам нужно будет использовать следующую формулу:

количество необходимых наборов рисунка = общая площадь проекта / (a ​​* b + c * d + e * f + g * h)

Некоторые популярные модели асфальтоукладчиков, которые могут вдохновить вас

Вот некоторые из самых популярных узоров, которые можно создать с помощью стандартных кирпичей для брусчатки размером 4 x 8 дюймов.

  • Спусковая связка — в основном связанная с простой кирпичной кладкой, беговая связка является одним из наиболее распространенных и наиболее естественных рисунков мощения. Его также можно создать из квадратного кирпича.

  • Многослойная связка — не очень часто используется с брусчаткой, но ее очень легко и просто создать. Может быть хорошей идеей, если вы укладываете брусчатку самостоятельно.

  • «Елочка» — на рисунке показан узор «елочка» под углом 90 °.45 ° не менее популярны.

  • Basketweave — плетение корзин создает иллюзию исчезающих друг под другом длинных кирпичей, как в настоящих корзинах. Существует бесчисленное множество вариаций этого вида рисунка брусчатки для патио.

Почему мощеный внутренний дворик — хорошая идея?

Есть ряд причин, по которым мы считаем, что укладывать брусчатку в патио — это отличная идея.

  1. Мощеные патио прочные .Брусчатку обычно делают из бетона, глиняного кирпича или даже натурального камня. Строительных материалов прочнее этого не так много. Брусчатка редко трескается, и можно ожидать, что она останется неизменной в течение многих лет.

  2. Хорошо себя чувствуют в переменную погоду . Твердые поверхности, такие как бетон, склонны к растрескиванию и, как правило, не очень хорошо себя чувствуют в местах, где температура резко меняется в зависимости от сезона. Это можно увидеть на многих дорогах — обратите внимание, сколько новых выбоин появляется после зимы.С брусчаткой это не проблема.

  3. Они неприхотливы в обслуживании . Как было сказано выше, брусчатка невероятно долговечна по сравнению с твердыми поверхностями, поэтому вам не придется делать много ремонта. Даже если вам все же придется сделать какой-то ремонт, обычно достаточно удалить крошечную часть патио, а не, скажем, целую плиту из бетона. Что касается чистки, часто достаточно использовать немного воды с мылом, чтобы патио выглядело как новенькое.

Деятельность: Садовая зона

Вы когда-нибудь задумывались, какова площадь вашего сада?

Давайте попробуем и узнаем!

Вам понадобится сад, рулетка, ручка и бумага… и твои мозги.

У меня нет сада, что мне делать?

Если у вас нет сада, возможно, он есть у друга или у ваших родственников, поэтому используйте их.

Насколько точными должны быть мои измерения?

Попробуйте измерить с точностью до сантиметра (или полдюйма), чтобы погрешность была как можно меньше.

Вы должны получить хорошую оценку, если будете осторожны при измерениях.

Есть ли простой способ найти участок моего сада?

Если ваш сад представляет собой прямоугольник, то у вас есть несложный расчет.Вам просто нужно измерить его ширину и длину и умножить их вместе:

Прямоугольник:
Площадь = Ш × Д

Но это делает это занятие слишком простым …
… так что ищите другой сад с более интересной формой!

У меня сад сложной формы, как мне найти его площадь?

Хорошо! Эта активность получила интересных

Это может быть одна из фигур на странице «Площадь плоских фигур», тогда вам просто нужно решить, какая форма, произвести измерения и использовать формулу.

Но вы также можете разбить сложную форму на треугольники:

Затем измерьте основание ( b ) и высоту ( h ) каждого треугольника:

Тщательно запишите каждое измерение, чтобы знать, к какому треугольнику оно принадлежит.

Теперь войдите внутрь и вычислите каждую площадь.
(используя Area = ½b × h) и сложите их все.

Но мой сад другой …

… на самом деле это вообще не какая-то форма … у него есть прямые края и несколько изогнутых частей. Что мне делать?

Может это выглядит примерно так:

Вы можете попробовать покрыть свой сад сеткой из квадратов — это могут быть квадраты размером 1 метр или 1 фут, примерно так:

Как сделать сетку?

Попробуйте воткнуть колышки в землю и соединить их веревкой.Убедитесь, что они находятся на правильном расстоянии друг от друга и все углы прямые.

Как это помогает? Сетка и контур сада не совпадают. Есть много углов и изогнутых деталей.

Подсчитайте квадраты!

Есть специальные методы, о которых говорится на странице Area. Самый простой способ:

  • более половины квадрата считается 1
  • меньше половины квадрата считается как 0

Оценка площади 41 м 2 .
(Это просто пример.
Ваш сад, конечно, другой.)

Если ваша сетка составляет 1 фут на 1 фут, то площадь будет в квадратных футах.

Зачем мне знать площадь моего сада?

Причин может быть много:

  • Вы хотите поменять газон в саду. Сколько травы нужно заказать? Сколько это будет стоить?
  • Вы хотите засадить сад томатами. Их нужно сажать на определенном расстоянии друг от друга.Сколько растений вы могли бы посадить? Какой будет ваш ожидаемый урожай помидоров?
  • Вы хотите устроить барбекю-вечеринку. Сколько людей может комфортно разместиться в вашем саду?

Теперь вы можете выполнить задание: трава для сада

Я закончил … что я узнал?

Вы узнали об измерении, записи данных, рисовании и вычислении площади, молодец!

Мебель для патио, Уличные зонтики для патио и рыночные зонтики, чехлы для мебели для вашего сада, лужайки или террасы.

Используйте навигационные ссылки слева или используйте поле поиска в правом верхнем углу.

  • САДОВЫЕ ЗДАНИЯ
  • Садовые беседки и арки
  • Садовые беседки и комплекты
  • Комплекты для теплицы
  • Навесы и комплекты для хранения
  • ГРИЛИ И НАГРЕВАТЕЛИ
  • Chimeneas
  • Грили — газовые, угольные, электрические
  • — Переносные настольные грили
  • Костровые ямы, кольца и камины
  • Обогреватели патио — Наружные
  • — Электрические лучистые обогреватели
  • — Инфракрасные обогреватели
  • НАРУЖНОЕ ОСВЕЩЕНИЕ
  • Садовые факелы
  • Пейзажное освещение
  • Уличные лампы
  • Патио / палубные фонари
  • Солнечное освещение
  • МЕБЕЛЬ ДЛЯ ПАЦИО
  • Алюминиевая мебель
  • Пляжная мебель
  • Мебель из эвкалипта
  • Гамаки и кресла-гамаки
  • Мебель для патио
  • Покрывала для патио
  • Мебель из поли-дерева
  • Качели для крыльца
  • Плетеная мебель из смолы
  • Кресла-качалки
  • Мебель из тика
  • Деревянная мебель для патио
  • ОТТЕНКИ И ТЕНТЫ
  • Только ткань

  • Выдвижные навесы
  • Roll Up Shade, Солнечные оттенки
  • Теневые навесы
  • Паруса тени
  • — Паруса нестандартного оттенка
  • НАБОРЫ И НАБОРЫ КАЧЕЛЕЙ
  • Наборы деревянных качелей с планами
  • Металлические качели
  • Принадлежности для качелей
  • Направляющие для качелей
  • БАССЕЙНЫ
  • Наборы для надземных бассейнов
  • Надувные лодки и шезлонги
  • Бассейны и оборудование
  • Инфракрасные сауны
  • ЗОНТИКИ
  • Алюминиевые зонты
  • Основания и подставки
  • Пляжные зонтики
  • Зонты с коммерческим логотипом
  • Ребристые зонты из стекловолокна
  • Зонтики для патио
  • Боковые пост-офсетные зонты
  • Деревянные зонтики
  • НАРУЖНЫЕ АКСЕССУАРЫ
  • Баня для птиц
  • Домики и лежаки для собак
  • Корпуса из искусственного камня
  • Садовый декор
  • Уличные часы и термометры
  • Уличные шторы / портьеры
  • Наружные вентиляторы — мокрые
  • Игры на открытом воздухе
  • Коврики для улицы — всепогодные
  • Уличные телевизоры — всепогодные
  • Экраны конфиденциальности патио
  • Принадлежности для задних дверей
  • Настенные фонтаны и водопады
  • НОВОСТИ И СТАТЬИ
  • Новости патио и сада
  • Предметы домашнего обихода на открытом воздухе

    Кредитные карты, которые мы принимаем!

    Обработка кредитной карты онлайн

  • & nbsp


    .

    Добавить комментарий

    Ваш адрес email не будет опубликован. Обязательные поля помечены *